GI Exam 2n(edit)

Réussis tes devoirs et examens dès maintenant avec Quizwiz!

What is the circulation of the liver?

1. (75%) of blood comes from the portal vein, while (25%) comes from the hepatic artery 2. Venous and arterial blood will mix in the Sinusoids 3. Mixed blood will be modified by the liver and then leave through the hepatic veins → Liver cells are never exposed to fully oxygenated blood → Liver cells are the first to received absorbed toxins and nutrients

Protozoa Life Cycle

1. Cysts: infectious form found in the external environment 2. Ingestion: Gastric acid stimulated excystation, and the protozoa will invade the wall of the colon, liver, & lungs where it can multiply and lead to ulcers (hepatitis & liver failure) 3. Within the GI system, the protozoa will multiply, forming a Trophozoite (metabolically active form) NOTE: the cyst and trophozoite can be diagnostic forms, but the cyst is the only infectious form

What are the different cells of the liver and their functions?

1. Hepatocytes: absorption of nutrients, secretion of proteins, & production of bile 2. Endothelial cells: Produce Factor 8 & create space of disse 3. Kupffer cells: Phagocytic macrophage of the liver 4. **Stellate/Ito cells: located in the space of Disse, that store Vitamin A and are involved in fibrosis which leads to cirrhosis**

Describe the physiology of swallowing

1. Oral Preparatory Phase: 5-10 seconds, voluntary control; Mastication, tasting, bolus forming 2. Oral Transport/ Propulsive Phase: 1 second, A/P bolus manipulation ends voluntary control of mechanism 3. Pharyngeal Phase: 1-2 seconds 4. Esophageal Phase: 10 seconds These phases overlap in succession and are not distinctly separated

Histological characteristics of the large intestine

1. Outer longitudinal layer of muscularis externa in the cecum and colon exhibits three thickened, equally spaced bands (teniae coli) 2. External surface of the colon exhibits sacculations (haustra) that are represented on the luminal surface as semilunar folds 3. Fat tissue deposited under the serosa forms small, fatty projections (omental appendices)

What are the structures contained in the portal triad?

1. Portal Vein 2. Hepatic Artery 3. Common Bile Duct

Explain the ducts of the liver/ gallbladder and how they connect

1. The liver contains both the right & left hepatic duct which join to form the common hepatic duct 2. The gallbladder contains the cystic duct 3. The common hepatic duct & the cystic duct will join together to form the common bile duct which will join with the pancreatic duct (from the pancreas) to dump into the sphincter of Oddi and into the duodenum

Postural (orthostatic) hypotension is indicative of what % blood loss and what type of bleed?

10-20% blood loss Moderate Bleed

Shock (resting hypotension) is indicative of what % blood loss and what type of bleed?

20-25% blood loss Massive Bleed

How much fiber is recommended for a patient experiencing constipation?

25 grams/day

Normal blood pressure is indicative of what % blood loss and what type of bleed?

<10% blood loss Minor bleed

Hepatocellular Adenoma

A benign tumor of the liver most frequently associated with oral contraceptives and found in adult woman Most often present with RUQ pain and have the potential for bleeding and malignancy Oral contraceptives should be stopped and resection is recommended is there is need for hormonal therapy, >4 cm, rapidly growing, or causing symptoms β-carotene positive stain, bile ducts are not present

Chronic Autoimmune Hepatitis Include Diagnosis, Symptoms, Laboratory findings, & Treatment

A chronic inflammatory disease of unknown etiology which is characterized by the presence of autoantibodies, hypergammaglobulinemia, and necroinflammatory changes on histology Diagnosis: Increase globulins, Increased autoantibodies, Biopsy shows interface hepatitis (lymphoplasmacytic infiltrate with bile ducts intact) Symptoms: Fatigue, Lethargy, malaise, arthralgia, anorexia, nausea, abdominal pain, dark urine Labs: Elevated AST & ALT (generally <500), hypergammaglobulinemia, Presence of ANA Treatment: Prednisone (monotherapy) or Combination therapy (Prednisone + Azathioprine)

Arteriovenous Malformation (AVM)

A collection of blood vessels with abnormal communication between the arteries and veins which can cause lower GI bleeding Increased risk in older patients, right sided location, and those with comorbid conditions (aortic stenosis and chronic renal failure) Diagnosis: Colonoscopy, Angiography

Whipple Disease Include description, signs/symptoms, and histological findings

A disease in which there is systemic tissue damage characterized by macrophages loaded with Tropheryma whippelii organisms Classic site of involvement is the small bowel, and it results in fat malabsorption & steatorrhea Signs/Symptoms: Steatorrhea, arthritis, cardiac valve involvement, and neurologic symptoms Histology: Partially destroyed organisms present in macrophage lysosomes, and stain positive for PAS

Stercoral colitis Characteristics, Presentation, and Treatment

A large amount of feces impacts and obstructs a narrow, angulated portion of the sigmoid colon, continuously pressing on the mucosa and leading to colitis and pressure necrosis Presentation: Ranges from crampy abdominal pain to colonic obstruction, and demonstrates radiologic evidence of colitis, endoscopic evidence of ulceration, and progression to necrosis and perforation CT will show colonic thickening and extraluminal bubbles Treatment: Manual disimpaction, endoscopic bowel disimpaction, Oral Laxatives are contraindicated and can lead to stercoral ulceration

Cirrhosis Define, Physical Findings, Radiographic Findings

A late stage of progressive hepatic fibrosis characterized by distortion of hepatic architecture and the formation of regenerative nodules In its advanced stages, it is considered irreversible Physical Findings: ∙ Spider Angiomata ∙ Nail Changes ("moon" extends almost entire nail bed) ∙ Dupuytren's Contracture ∙ Fetor Hepaticus (distinct smell of breath) ∙ Jaundice ∙ Asterixis (hand flaps when held up) ∙ Ascites Radiographic Findings ∙ Liver may appear small or nodular ∙ 99mTc sulfur colloid in nucleotide studies will be taken up by spleen and bone marrow

Dieulafoy's Lesion

A rare cause of upper GI bleed caused by a large tortuous arteriole in the stomach wall that erodes and bleeds. It can cause gastric hemorrhage Often difficult to identify endoscopically because it pulsates and shows, but then returns into the mucosa

Hepatic cystadenoma

A rare neoplasm occurring in middle-aged women containing cystic structures within the lesion. Abdominal pain is common and often the presenting symptom-- they can rupture, necrose, or calcify and undergo malignant transformation Should be removed when possible

AST

A transaminase that catalyzes the transfer of the amino group from alanine and aspartic acid to alpha-ketoglutaric acid Found in the liver, heart, skeletal muscle, kidney, brain, pancreas, lung, leukocytes, and erythrocytes Because its widespread distribution, it has low specificity for any single disease Elevation suggests inflammation (located in the mitochondria)

ALT

A transaminase that catalyzes the transfer of the amino group from alanine and aspartic acid to alpha-ketoglutaric acid Located primarily in liver and kidney, with lesser amounts in heart and skeletal muscle Increased ALT activity is more specific for liver damage than increased aspartate aminotransferase (AST) activity. ALT is seldom increased in patients with heart or muscle disease in the absence of liver involvement Elevation suggests inflammation (located in the cytoplasm)

HELLP Syndrome

A variant of gestational hypertension (severe form of eclampsia) Hemolysis, Elevated Liver enzymes, Low Platelets Preceded by RUQ pain, chest pain, & peritoneal signs

In what form does fat first get absorbed in the digestive tract? A. 2-monoacylglycerol + fatty acids B. Chylomicrons C. Diacylglycerol + fatty acids D. Fatty acids + glycerol E. Phospholipids F. Triglycerides

A. 2-monoacylglycerol + fatty acids

Define the GI Bleed: A. Upper B. Small Bowel C. Lower

A. Above the Ligament of Treitz B. AKA Mid-intestinal; from the ligament of Treitz to the ileocecal valve C. Lower: Below the ileocecal valve

A 26-year-old female went camping in Colorado. During her trip, she consumed water from a lake inhabited with beavers. Upon return, she developed severe foul smelling diarrhea. Stool O&P was positive for Giardia. What are the clinical benefits and pitfalls of each drug? A. Metronidazole B. Tinidazole C. Paromomycin D. Nitazoxanide

A. Cannot use in pregnancy or in combination with alcohol B. Longer acting, shorter duration of treatment than Metronidazole, but extremely expensive; still cannot use in pregnancy or with alcohol C. Very safe and tolerable, can use in pregnancy D. Expensive, but readily available with fewer side effects than Paromomycin

What are the findings in the following when there is obstructive Jaundice A. ALP B. Bilirubin C. Large bile ducts D. Small bile ducts E. Histology

A. Elevated B. Elevated C. Dilated D. Proliferation E. Bile lakes

What are the findings in the following when there is Intrahepatic Cholestasis A. ALP B. Bilirubin C. Large bile ducts D. Small bile ducts E. Histology

A. Elevated B. Normal-Elevated C. Normal D. Normal- Abnormal E. Bilirubin stasis

If diarrhea occurs in the following time frames, what is the problem? A. Quick B. Longer C. Chronic

A. Enterotoxin (intoxication) B. Bacterial growth (infection) C. Persistent = Parasite

What is the likelihood of liver injury from a new medication administration if the following labs are obtained? A. Serum ALT < 3x upper limit of normal B. Serum ALT 3-5x upper limit of normal C. Serum ALT >5x upper limit of normal

A. Frequent, not associated with liver injury, may not lead to liver injury B. 10% risk of developing acute liver failure if patient becomes jaundice → medication should be stopped C. Up to 50% risk of developing liver failure if patient becomes jaundice → medication should be stopped

What is true of most amino acid transporters? (select all that apply) A. Overlapping specificity for substrates B. Require ATP C. Sodium-dependent cotransport D. Unidirectional

A. Overlapping specificity for substrates C. Sodium-dependent cotransport Will not require ATP directly (allow transport by allowing Na+ to come down its concentration gradient) & have to ability to move bidirectionally

What are the two main parasite classifications?

A. Protozoa: single-celled organisms that want to give you diarrhea to spread out of the body B. Metazoa: Multicellular organisms that lay eggs and want to stay undetected; can live in GI tract for years

A 45-year-old male presents to the ED with progressively worsening lethargy, confusion, myalgias, fever and chills over the last few months. Medical history includes HIV. Further questioning reveals that he was consuming undercooked pork from a street vendor in in South America a few years ago while on vacation. CT of his brain demonstrates significant cystic lesions throughout his brain parenchyma. A. What is the underlying diagnosis and causative agent? B. What is the treatment of choice? C. What is the drug's MOA? D. What are you monitoring during treatment?

A. Taenia Solium- Neurocysticercosis B. Albendazole C. Inhibits microtubule synthesis and glucose uptake of nematodes D. Seizures, Liver status (can cause acute liver injury), CBC (can cause bone marrow suppression)

A 53-year-old male presents to you for evaluation of waxing and waning cough, abdominal pain and diarrhea that has been ongoing for weeks. His symptoms starting after he noticed pink, erythematous and serpiginous tracts on his feet that were extremely pruritic. He is from a rural town in China and has recently immigrated to the USA. Basic lab work reveals a high peripheral eosinophil count. A. What is the most likely diagnosis? B. What is the drug of choice? C. What is the MOA of the DOC?

A. Threadworm (Strongyloides stercoralis) B. Ivermectin C. Binds chloride channel, chloride influx is enhanced and hyperpolarization occurs which results in paralysis

Identify the pharyngeal recesses.

A. Vallecula: space between the epiglottis and base of the tongue B. Pyriform Sinuses: Bordered medially by the aryepiglottic and inferiorly by the cricopharyngeal m. Benefits: Allows food and liquid to hesitate, recesses create a moat around the opening to the trachea Risks: Residue in the pharyngeal recesses may not clear, and can spill over resulting in aspiration

A 43-year-old female has just undergone her first treatment with cisplatin, a chemotherapy agent, to treat her ovarian cancer. Which of the following medications is most likely to improve her symptoms of nausea and vomiting? A. ondansetron B. metoclopramide C. dronabinol D. scopolamine E. meclizine

A. ondansetron

Non-digestible carbohydrates

AKA Fiber (found in fruits, vegetables, and whole grains) Responsible for normalizing bowel movements, aiding in digestion and feeling satiated, slow sugar absorption thereby regulating insulin response May also help lower LDL

Hereditary non-polyposis colorectal cancer (HNPCC)

AKA Lynch Syndrome It is an Autosomal dominant disease, due to a mutation in MSH2/MSH1 gene, which results in colorectal cancer with fewer polyps than in FAP

Microfold Cells

AKA M-cells The are modified enterocytes which cover the lymphatic follicles (Peyer's patches) of the small intestine They have a squamous appearance with widely spaced villi, and will absorb macromolecules from the lumen, discharge molecules to the lymphocytes, and stimulate lymphocytes to secrete IgA

Relation between ALT & AST in liver injury

ALT > AST

Is AST or ALT specific to the liver?

ALT is predominant to the liver AST is located not only in the liver, but skeletal muscle, heart, brain, and kidney

Relation between ALT & AST in liver cirrhosis

AST > ALT

Relation between ALT & AST in alcoholic liver

AST > ALT (2:1)

Describe the absorption of: vitamin B12 Including location, as well as enzymes and transporters involved.

Absorption: Absorbed in the ileum, and requires intrinsic factor

Describe the absorption of: Iron Including location, as well as enzymes and transporters involved.

Absorption: Only small amounts are absorbed daily in the duodenum and jejunum Tea and milk can interfere with absorption transferrin is required for its transport

Enterocytes

Absorptive (sugars, amino acids, lipids) epithelial cells that are found in the small intestine. They also aid in secreting IgA made by the plasma cells. They are columnar cells with microvilli that will have an ovid nucleus with numerous, elongated mitochondria

Stress Ulcers

Acute gastric mucosal lesions occurring after medical crisis or trauma which lead to upper GI bleedings A. Curling Ulcer: occur following large body surface burns B. Cushing Ulcer: occur following central nervous system injury

What is the number one cause of liver failure in the united stated?

Alcohol

What liver tests are associated with cholestasis?

Alkaline phosphatase Isoenzymes: Specific for liver pathology, elevated levels correlate with cholestasis, cirrhosis, liver metastasis Gama glutamyltransferase: membrane-bound peptidase that hydrolyzes peptides to amino acids and smaller peptides. Particularly sensitive to alcohol consumption, and can be elevated even when other liver function tests are normal Bilirubin: When RBC's die, Hemoglobin is released and is converted to unconjugated (also called indirect) bilirubin. In the liver, unconjugated bilirubin is transferred from the albumin into hepatocytes where it is conjugated (direct)

Compare & contrast between: A. Hernias B. Adhesions C. Volvulus D. Intussusception

All cause intestinal obstruction-- most often the small intestine A. Any weakness or deficit in the abdominal wall that permits the protrusion of a serosa lined pouch of peritoneum. They are the most frequent cause of intestinal obstruction worldwide B. The most common cause of small bowel obstruction, they are fibrous bridges between bowel segments that occur typically after surgical procedures, peritoneal infection, etc. C. Complete twisting of an intestinal loop which can result in bowel strangulation and infarct. Most often occurs in the sigmoid colon (elderly) and cecum (young adults) D. When a segment of intestine, constricted by a wave of peristalsis, telescopes into the immediate distal segment. Once trapped, the invaginated segment is propelled by peristalsis and pulls the mesentery. In children it is associated with rotavirus. In adults, it is associated with tumors. Can lead to infarct due to disruption of blood supply. It is the most common cause of intestinal obstruction in children (3 month-6 years)

Celiac Disease Include Signs/Symptoms, Diagnosis, and Treatment

An autoimmune disease that results in the immune system attacking the micro-villi of the small intestine, and causing further damage to vili when patients continue to consume gluten. Causes less absorptive area. Damage is most prominent in the duodenum Signs/Symptoms: No classic signs, but can cause bloating, abdominal pain, and diarrhea; patients may have HLA-DQ2 allele Diagnosis: IgA Antibodies, anti-endomysial antibodies (AEA), and anti-tissue transglutaminase antibodies (t-TG) can be found. Duodenal biopsy shows flattening of villi, hyperplasia of crypts, and increased intraepithelial lymphocytes Treatment: Strict gluten-free diet

Celiac Disease

An immune reaction to eating gluten which results in small intestine damage, leading to destruction of the villi and mucosal inflammation, decreasing absorption in the small intestine This causes diarrhea, bloating/gas, fatigue, and anemia To diagnose, serum IgA antibodies to gliadin, tTG, EMAs can be taken or genetic testing for HLA-DQ2, DQ8 can be done

Paneth Cells

Antimicrobial secreting epithelial cells of the small intestine They will contain basophilic basal cytoplasm with acidophilic secretory granules on the apex The granules will contain lysozyme (digests bacterial cell wall), α-defensins (mediator in CD8 lymphocytes), an arginine-rich protein (responsible probably for acidophylia) and zinc.

AIMS65 Score

Assesses risk of Upper GI mortality (the higher the number, the greater the mortality rate) 1. Albumin < 3.0 mg / dL 2. INR > 1.5 3. Mental status change (GCS < 15) 4. Systolic blood pressure < 90 5. Age > 65 years

Describe familial adenomatous polyposis (FAP)

Autosomal dominant disorder characterized 1,000s of adenomatous colonic polyps Due to inherited APC mutation Colon and rectum are removed Prophylactically, otherwise almost all patients will develop carcinoma by 40 years of age Histologically will note polyposis overgrowth

Juvenile Polyposis Syndrome (JPS)

Autosomal dominant syndrome, characterized by 3-100 hamartomatous polyps and often associated with dysplasia. Both BMPR1A and SMAD4 have been linked to the syndrome. Patient's with the SMAD4 mutation are also at risk of having hereditary hemorrhagic telangiectasia (localized rupture of thin-walled blood vessels, especially around the mouth and GI tract) Most patients develop colonic adenocarcinoma by age 45

What are the pathology, clinical findings, laboratory features of, & Treatment: Wilson's Disease

Autosomal recessive disorder of copper metabolism characterized by excess deposition of copper in the liver, brain, and other tissues Pathological: Intestinal copper is absorbed and transported to the hepatocytes, the transport of copper by copper transporting enzymes into the bile is defective , and excess copper deposition promotes free radical formation Clinical: gene mutation (ATP7B) leading to defective transport of copper into bile and unbound copper accumulating in the blood (Liver > Eye > Brain); Kayser-Fleischer ring Laboratory: Elevated 24-hour urinary copper excretion & decreased serum ceruloplasmin Treatment: D-Penicillamine + Trientene will chelate copper, Zinc will block absorption of copper

Which enzyme complex contains catalytic activity towards lactose? A. Beta-glucoamylase B. Beta-glycosidase C. Sucrase-isomaltase D. Trehalase

B. Beta-glycosidase

Which 2 monosaccharides are linked to form lactose? A. Galactose + Fructose B. Galactose + Glucose C. Glucose + Fructose D. Glucose + Glucose

B. Galactose + Glucose

Describe the inter-relationship between folate and vitamin B12, in particular with regard to the "methyl trap" ("folate trap") (Marks pp. 798-810)

B12 is required to transfer a methyl group from the inactive form of folate (N5-methyl-FH4) to Homocysteine to produce the active form of folate (FH4) and Methionine If there is a Vitamin B12 deficiency, there will subsequently be a deficiency in folate

State the difference between a Barium Swallow and a Modified Barium Swallow Study.

Barium Swallow: The patient swallows barium liquid which coats the esophagus or stomach and a series of x-rays are taken to track its path through the digestive system. Barium swallow test shows where foods and liquids go when a person swallows Modified Barium Swallow: completed to determine the physiology of swallowing; not performed to determine the presence or absence of aspiration or to recommend a diet; patients are seated in front of a fluoroscopy, or X-ray, machine operated by a radiologist & Foods are coated with barium to allow for viewing under the fluoroscopy machine. The patient then swallows the food or liquid, and medical staff look for problems during the oral and esophageal phases of swallowing

How is the histologic structure of the vermiform appendix different from the colon?

Basically similar, but with the following changes: 1. Small lumen 2. No Tenia 3. Excessive amount of lymph follicles in the lamina propria and submucosa

Gilbert's Syndrome

Benign congenital unconjugated hyperbilirubinemia which can be reduced with phenobarbital The most common cause of jaundice in an otherwise healthy person, it is due to reduced conjugation of bilirubin Patient's jaundice will worsen during fasting

Focal nodular hyperplasia (FNH)

Benign liver mass composed of a combination of hepatocytes and fibrous tissue that typically contains a central, stellate scar Typically asymptomatic and an incidental finding Bile ducts are present

Hepatic hemangioma

Benign, most common liver tumor which has the potential of bleeding, thrombosing, necrosing, or calcifying usually asymptomatic, and symptoms that do occur are due to mass effect Treatment is symptomatic

What are the causes of unconjugated and conjugated hyperbilirubinemia

Bilirubin is the end product of heme degradation. Unconjugated (indirect) bilirubin is lipophilic and will bind albumin. Hepatocytes turn unconjugated bilirubin into conjugated (direct) bilirubin which is water soluble and can be excreted via the gut and urine Unconjugated hyperbilirubinemia: unable to conjugate the bilirubin A. Gilbert's: mild deficiency of glucoronyl transferase therefore cannot conjugate bilirubin B. Crigler-Najjar Syndrome Type I: severe deficiency of glucoronyl transferase therefore cannot conjugate bilirubin C. Crigler-Najjar Syndrome Type II: moderate deficiency of glucoronyl transferase therefore cannot conjugate bilirubin Conjugated hyperbilirubinemia: bilirubin can be conjugated, but is unable to be transported into the bile

Obscure Bleeding

Bleeding is clinically obvious, but the source is note easily identified on routine examination

Occult Bleeding

Bleeding is not apparent by the patient and is caused by small amounts of bleeding

Post-Polypectomy Bleeding

Bleeding seen following polypectomy (removal of polyps), usually within 7 days but can be up to 20 days Risk is increased with anticoagulants

Intrahepatic Cholestasis

Blockage of intrahepatic bile ducts caused by: drugs, toxins, Alcohol, hepatic metastasis ALP translocated to the basolateral membrane of the hepatocyte A. ALP: Elevated B. Bilirubin: Normal-Elevated C. Large bile ducts: Normal D. Small bile ducts: Normal-Abnormal E. Histology: Bilirubin Stasis

Budd-Chiari Syndrome

Blockage of the hepatic vein (between the liver and inferior vena cava) stopping the flow of blood out of the liver and back to the heart resulting in an engorged/swollen liver. In pregnancy there is a hypercoagulable state which can lead to this syndrome. Can cause ascites, hepatomegaly, abdominal pain, & Jaundice

Differentiate between a co-infection and superinfection with HBV and HDV, and the outcomes of both

Both are in IV drug users Co-Infection: Infected with Hepatitis B and Hepatitis D viruses at the same time. Hepatitis B must first establish infection, requiring Hepatitis D to wait. Less severe than superinfection Superinfection: Patient infected with Hepatitis D after Hepatitis B infection, allowing Hepatitis D to replicate immediately. Produces a more rapid and severe progression

65-year-old male is brought to the hospital with jaundice. A blood test reveals that serum levels of direct (conjugated) bilirubin are elevated. Serum levels of urobilinogen are low, but present. Urine samples show elevated levels of bilirubin. What is the most likely cause of the jaundice? A. Obstruction of the hepatic artery B. Partial fibrotic obstruction of the liver biliary ducts due to cirrhosis C. Complete obstruction of the common bile duct D. Mutation of the UGT enzyme E. Red blood cell hemolysis

C. Complete obstruction of the common bile duct Conjugated bilirubin is found in the hepatocyte. Therefore, there must be a blockage in getting the bile to the intestine, and therefor blockage of the common bile duct

An 8-year-old boy presents to your office for a new well-child visit. During your physical examination you notice his teeth are discolored. What medication did his mother most likely take during her pregnancy with the patient? A. Bactrim B. Metronidazole C. Doxcycline D. Ivermectin E. Tinidazole

C. Doxcycline

A 75-year-old female with documented diabetic gastroparesis presents to the office for follow-up. She has noted good blood glucose control over the past 3 months, but still complains of early satiety and nausea. Prior to beginning treatment with metoclopramide what is the most severe adverse effect of the medication that should be discussed with her? A. Ischemic colitis B. Prolongation of the QT interval C. Tardive dyskinesia D. Hyperkalemia E. Diarrhea

C. Tardive dyskinesia

Discuss the metabolic functions of the liver, including carbohydrate, fat metabolism and the storage functions of the liver

Carbohydrate: When blood glucose levels are low, it is responsible for converting glycogen to glucose, and when blood glucose levels are high, it is responsible for converting glucose to glycogen for storage. It also has the ability to form glucose using amino acids and fatty acids Lipids: Liver will metabolize fats through beta-oxidation to form acetyl-coA, which can then be circulated in the blood to different organs and tissues. 80% of cholesterol in the body will be converted into bile salts by the liver, which can then be secreted into bile Protein: Will form non-essential amino acids, deamination of amino acids and formation of keto-acids, as well as the excretion/ formation of urea (made from ammonia)

Globulins

Carrier proteins made by the liver which transport specific substances; acute phase reactants, which are involved in the inflammatory process; coagulation factors; complement components; and immunoglobulins

Lactose Intolerance

Caused by a deficiency in lactase, there is a build up of excess lactose because the β-1,4-bond is unable to be cleaved The Lactose will stay in the intestinal lumen, causing an uptake of water into the lumen, and digestion by gut bacteria which leads to production of H2 and methane gases causes flatulence and abdominal discomfort

What are the causes (i.e. name of bacteria/virus /etc.), morphologic findings, & clinical features of infectious enterocolitis?

Causes: Clostridium Difficile (Pseudomembranous colitis), Cholera (bacterial enterocolitis) Morphologic Findings: ∙Pseudomembranous Colitis- superficial, graying mucosal exudates consisting of necrotic, loosely adherent mucosal debris. Histologically will note neutrophil infiltration (looks like volcanic eruption) with fibrin cap ∙Bacterial Enterocolitis- Gram (-), it is transmitted via the fecal-oral route through contaminated food and water which produces an enterotoxin that will disrupt the cells causing loss of fluid and electrolytes; mucosa will look normal on biopsy Clinical Features: ∙Pseudomembranous Colitis- fever & diarrhea, most often occurring in patients on broad-spectrum antibiotics ∙Bacterial Enterocolitis- watery diarrhea

ALP + GGP elevation is indicative of what?

Cholestasis

Describe the mechanism of bile secretion, including the physiological anatomy of biliary secretion, how bile is stored in the gallbladder, the specific composition of bile, and the factor that stimulates the emptying of the gallbladder

Cholesterol is converted into primary bile acids (Cholic Acid & Chenodeoxycholic Acid) The Primary bile acids are then converted into secondary bile acids (Deoxycholic Acid & Lithocholic Acid). Secondary bile acids will be conjugated to Glycine, Taurine, etc. to form bile salts If less bile acid is reabsorbed (ileum resection), rate of synthesis is increased; if excessive bile acids are absorbed, the rate of synthesis will decrease Bile Transport: A. NTCP: Sodium-dependent Taurocholate Co-transporting polypeptide; allows enter of bile acids into the hepatocyte B. OATP: Multi-organic anion transporting protein; allows bile/ bilirubin into the hepatocyte → Metabolism of hemoglobin generates bilirubin, when bilirubin is not conjugated (bound to something) it is extremely toxic, and therefore is typically conjugated to glucuronic acids within the hepatocyte by UGT (UDP glucuronyl transferase) C. BSEP: Bile salt export protein; allows for secretion of bile salts into canaliculi Storage Stored in the gallbladder, until it is secreted into the intestine, where it will be unconjugated and metabolized by bacteria, and 90% (stercobilin) excreted in the feces while 10% is returned to the plasma

Primary Sclerosis Cholangitis (PSC) Include Risk Factors, Diagnosis, Histological Findings, & Treatment

Chronic cholestasis associated with chronic inflammation of the biliary epithelium (large bile ducts affected) which results in fibrosis, cirrhosis, and malignancy Risk Factors: Obliterating fibrosis of intrahepatic and extrahepatic bile ducts, most often associated with ulcerative colitis Diagnosis: On X-ray imaging there is diffuse, multifocal strictures and focal dilation of the bile ducts, leading to a beaded appearance Histology: Onion skin like fibrosis Treatment: Endoscopic Treatment -- expansion of dominant strictures by dilation & Liver Transplant ◦ Cancer Surveillance: bile duct, gallbladder, colorectal

Neostigmine, Physostigmine Class, Indication, & Side effects

Class: Acetylcholinesterase inhibitor (pro-kinetic agent) Indication: Paralytic Ileus Side Effects: Massive diarrhea (do not give for nausea), bradycardia, flushing, cramps, salivation, blurred vision

Promethazine (Phenergan) Class, Indication, Adverse Effects, Contraindications

Class: Antihistamine (H1 receptor antagonist) Indication: Prophylactic treatment of Motion sickness/ dizziness, N/V associated with anesthesia & surgery Adverse Effects: Gangrene, CNS depression/ drowsiness, Respiratory failure, non-specific QT changes on EKG Contraindication: Pregnancy

Meclizine (Antivert) Class & Indication

Class: Antihistamine (H1 receptor antagonist) Indication: Vertigo, motion sickness

Scopolamine Class, Indication, MOA, Side effects, and Contraindications

Class: Antimuscarinic/ Anticholinergic Indication: prophylaxis again motion sickness; when nausea is stimulated by the vestibular apparatus (motion sickness) MOA: M1-Muscarinic receptor antagonist; selective on cortex and vestibular apparatus Side effects: CNS depression, dry mouth, restlessness, irritability, disorientation Contraindications: Glaucoma, Enlarged prostate

Pepto bismol Class, MOA, Indications, & Adverse reactions

Class: Bismuth Subsalicylate MOA: Antisecretory and antimicrobial action Indications: Non-specific diarrhea, traveler's diarrhea (ETEC) Adverse reactions: Black stool

Prochlorperzine (Compazine) Class, MOA, Indications, Side effects, & contraindications

Class: Dopamine Receptor Antagonist MOA: Blocks D2 receptors in the CTZ & M1 and H1 receptors Indications: N/V associated with anesthesia & surgery, active and prophylactic treatment of motion sickness, nausea associated with migraines, chemotherapy induced (chemical irritants) emesis, used as primary ant-emetic Side Effects: EPS, Tardive Dyskinesia, Increased prolactin (amenorrhea, gynecomastia), seizures, dementia-related psychosis Contraindications: Pregnancy

Domperidone (Motilium)

Class: Dopamine Receptor Antagonist MOA: Peripheral antagonist of D2 receptor Indications: N/V secondary to treatment with levodopa in Parkinson's disease Used in studies outside of US due to toxicity

Metoclopramide (Reglan) Class, MOA, Indications, Side effects, & contraindications

Class: Dopamine Receptor Antagonist MOA: True prokinetic- enhances coordinated transit in the gut (upper GI); improves/ starts peristalsis Indications: *Gastroparesis,* Nausea in pregnancy, persistent hiccups Adverse Effects: CNS depression, EPS, Tardive Dyskinesia (lip smacking), Increased prolactin (amenorrhea, gynecomastia), Prolonged QT interval

Vitamin D Class, Active Form, function, sources, and disease of deficiency

Class: Fat Soluble Active Form: Calcitrol, Cholecalciferol Function: Regulate plasma Ca2+, phosphate, & Parathyroid hormone levels Sources: Dairy products/ sunlight/ liver/ liver fish oils Deficiency: ∙ Rickets → in children, it is incomplete calcification of the bones; pigeon breast deformity, bowing of legs ∙ Osetomalacia → in adults, it is demineralization of the bone ∙ Renal Failure → chronic renal failure with decreased ability to produce Calcitrol ∙ Hypoparathyroidism → failure of PTH production leads to hypocalcemia & hyperphosphatemia

Vitamin K Class, Active Form, function, sources, and disease of deficiency

Class: Fat Soluble Active Form: Phylloquinones, Menaquinones Function: Blood clotting- synthesis of active clotting factors II/VII/IX/X Sources: Leafy vegetables/ intestinal bacteria/ liver/ egg yolk Deficiency: Failure of blood clotting → prolonged coagulation time, conjunctival & nail bed pallor, subcutaneous ecchymosis, bleeding gums, hematemesis

Vitamin A Class, Active Form, function, sources, and disease of deficiency

Class: Fat Soluble Active Form: Retinol, β-carotenes (approx. 1 year supply in liver) Function: Vision, Growth & differentiation, reproduction, Maintenance of epithelial cells Sources: Plants (especially carrots)/ Liver/ Fish/ Fish liver oils Deficiency: ∙ Night Blindness → diminished visual acuity primarily at night ∙ Xerophthalmia → severe deficiency which if untreated leads to blindness

Vitamin E Class, Active Form, function, sources, and disease of deficiency

Class: Fat Soluble Active Form: Tocopherols Function: Antioxidant with possible preventative role in atherosclerosis, CAD cancer, cataracts, Alzheimers; appears to enhance heme synthesis Sources: Green leafy vegetables/ vegetable oils/ liver/ eggs Deficiency: Mild Hemolytic Anemia → destruction of RBC faster than they are made

Linaclotide (Linzess) Include Class, MOA, Indications, Adverse Effects, & Contraindications

Class: Guanylate Cyclase Agonist MOA: Increases cGMP to stimulate Cl- and HCO3- secretion through the CFTR ion channel Indications: Chronic idiopathic constipation, IBS-C Adverse Effects: Diarrhea, Abdominal pain, Flatulence, Headache Contraindications: Mechanical GI obstruction in Pediatric Patients

Erythromycin Class, MOA, & Indications

Class: Macrolide antibiotic MOA: Motilin-like agent Indications: Short term diabetic gastroparesis, Upper GI bleed

Aprepitant/Fosaprepitant (Emend) Class, Indication, & Adverse Effects

Class: Neurokinin Receptor Antagonists (NK-1) Indication: N/V associated with emetogenic chemotherapy drugs (prevent acute and delayed emesis) Adverse Effects: Fatigue, Nausea, Constipation, Weakness, Hiccups

Loperimide Class, MOA, Indications, Adverse Effects, & Contraindications

Class: Opioid-receptor agonist MOA: Inhibits peristalsis and prolong transit time through the opioid receptor on circular and longitudinal intestinal muscles; allows the body to recoup water and electrolytes Indications: Acute or chronic diarrhea Adverse effects: Constipation Contraindications: used with caution in treatment of AIDS patients (can cause toxic megacolon)

Methylnaltrexone (Relistor)

Class: Selective Peripheral Opioid Receptor Antagonist MOA: Works on the gut opioid receptor, blocking it from opioids interacting with it Indications: Opioid Induced Constipation Adverse Effects: Abdominal pain, Flatulence, Nausea, Dizziness, Bowel Perforation

Vitamin C Class, Active form, function, sources, and disease of deficiency

Class: Water Soluble Active Form: Ascorbic Acid Function: Acts as an antioxidant, required for collagen synthesis, production of noradrenaline, and absorption of iron Sources: fresh fruit (especially citrus) & vegetables (especially tomatoes) Deficiency: Scurvy → sore, spongy gums, fragile blood vessels, loose teeth, splinter hemorrhages of the nail beds & poor wound healing

Vitamin B7 Class, function, sources, and disease of deficiency

Class: Water Soluble Active Form: Biotin Function: coenzyme responsible for several carboxylation reactions (CO2 fixing enzymes) Sources: Eggs/organ meats/ milk Deficiency: Rare due to plentifulness & strong binding of enzymes → can be induced by excess egg whites (i.e.-egg nog) which contains Avidin that binds biotin-- will cause nausea, dermatitis, alopecia

Vitamin B9 Class, Active form, function, sources, and disease of deficiency

Class: Water Soluble Active Form: Folic Acid Function: Coenzyme which is required for cell growth & replication Sources: Green leafy vegetables/ Liver Deficiency: Megaloblastic anemia & in offspring neural tube defects

Vitamin B5 Class, Active form, function, sources, and disease of deficiency

Class: Water Soluble Active Form: Pantothenic Acid Function: Essential component of Coenzyme A and Fatty Acyl Synthetase which are needed for a number of metabolic processes (i.e.- synthesis of triacylglycerol and phospholipids) Sources: Organ meats/ eggs/ legumes/ whole grains Deficiency: None documented

Vitamin B6 Class, Active form, function, sources, and disease of deficiency

Class: Water Soluble Active Form: Pyridoxal phosphate Function: Acts as coenzymes for numerous reactions (most involving amino acids- including synthesis and degradation) including heme synthesis Sources: Liver/ mackerel/ egg yolk Deficiency: Microcytic hypochromic anemia → deficiency in first step of heme synthesis

Vitamin B2 Class, Active form, function, sources, and disease of deficiency

Class: Water Soluble Active Form: Riboflavin Function: Forms coenzymes (FAD & FMN) required for redox reactions including the respiratory chain Sources: milk/ eggs/ organ meats/ leafy vegetables Deficiency: Araboflavinosis → sore throat, inflammation of skin & tongue, cheilosis (inflammation of corners of mouth), seborrheic dermatitis (scaly skin) especially of nose and scrotum

Vitamin B1 Class, Active form, function, sources, and disease of deficiency

Class: Water Soluble Active Form: Thiamine Pyrophosphate Function: Oxidative decarboxylation reactions & Transketolase reactions both of which are required for energy production & play a role in nerve conduction Sources: unrefined cereals/ legumes/ meat/ nuts Deficiency: ∙ Beriberi → commonly found in chronic alcoholics, there are two forms: 1. Dry- Peripheral polyneuropathy 2. Wet- Cardiomyopathy, Edema, & Heart dysfunction ∙ Wernicke-Korsakoff Syndrome → confusion, ataxia, memory loss, & hallucinations found in alcoholics d/t decreased absorption of thiamine

Vitamin B12 Class, Active Form, function, sources, and disease of deficiency

Class: Water Soluble - NOTE not found in plant tissue Active Form: Cobalamin Function: Absorbed in the ileum, it requires three glycoproteins (haptocorrin, intrinsic factor, and transcobalamin) and is responsible for methylmalonyl CoA Mutase and Methionine synthase reactions Sources: Liver (where it is stored)/ eggs/ intestinal bacteria Deficiency: ∙ Pernicious Anemia → not enough RBC produced ∙ Neurological Disorders → deficiency in synthesis of normal myelin d/t the accumulation of Methylmalonyl coA and the inability to convert it to Succinyl CoA to be passed into the TCA cycle

Vitamin B3 Class, Active form, function, sources, and disease of deficiency

Class: Water Soluble; note- not strictly a vitamin since it can be produced on a very high protein diet (produced from tryptophan) Active Form: Niacin Function: Forms coenzymes (NAD+ & NADP+) which function in redox reactions, also raises HDL and lowers LDL Sources: Organ meats/ nuts/ legumes Deficiency: Pellagra (typically seen in alcoholics & less frequently in GI disorders or the elderly) → Presents with the "3 D's": Dermatitis, Diarrhea, & Dementia

Diphenoxylate + Atropine (Lomotil)

Class: opioid-receptor agonist MOA: Inhibits excessive GI motility and GI propulsion; chemically related to the narcotic drug meperidine Indications: Diarrhea Adverse effects: Physical & psychological dependence, dry mouth, sedation Contraindications: Use in caution with children

Difenoxin + Atropine (Motofen)

Class: opioid-receptor agonist MOA: Inhibits peristalsis and prolong transit time through the opioid receptor on circular and longitudinal intestinal muscles; allows the body to recoup water and electrolytes Indications: Diarrhea, IBS, Adverse effects: Physical & psychological dependence, CNS depression, Nausea/ Vomiting, Dry mouth Contraindications:

Submucosa of the small intestine

Composed of dense, irregular connective tissue, it will contain the Meissner autonomic plexus In the Duodenum, it will contain mucinous glands (Brunner's glands), and in the ileum, it will contain lymphoid follicles (Peyer's patches)

Acute Alcoholic Hepatitis Include Laboratory tests & Treatment

Condition resulting from prolonged use of alcohol, in which the liver is inflamed; can be fatal Patients typically have a history of heavy alcohol use (years to decades) and recent abstinence. The may presents with jaundice, ascites, fever, and tender hepatomegaly Laboratory Test ∙ AST > ALT (2:1); AST < 400 → if higher, suspect concomitant use of acetaminophen ∙ Prolonged INR & Prothrombin time ∙ Decreased platelet count ∙ Leukocytosis (WBC <10,000, PMN >6,000) Treatment ∙ Manage intoxication & withdrawal ∙ Nutrition ∙ Manage metabolic derangements: Electrolytes (Mg, PO4, K), thiamine, glucose, folate, Vitamin K ∙ Abstinence ∙ Prednisolone 40mg/day x 28 days for DF (discriminant function)>32

Crigler-Najjar Syndrome

Congenital unconjugated hyperbilirubinemia due to either absent (Type I) or decreased (Type II) UDP GT activity Excess bilirubin can be uptaken by the brain and cause mental retardation

Compare & contrast the morphologic findings & clinical features of Crohn disease & ulcerative colitis

Crohn's Disease: Chronic Inflammatory condition that can occur anywhere in the GI tract, but most commonly found in the Ileum, Ileocecal valve, and cecum. NOD2 mutation thought to play a role ∙Morphologic Findings: Chronic inflammation involving all layers of the intestinal wall, thickening of involved segments with narrowing lumen, linear ulcerations, segmental with uninvolved bowel separating areas of involved bowels (Skip Lesions). X-rays will show string sign (severe narrowing of bowel) Histology shows involvement of all layers with non-caseating granulomas ∙Clinical Features: Transmural inflammation of the bowel wall, non-caseating granulomas, potential for fissures and fistulas between loops of bowel and other structures; inflammation is typically Abdominal pain, diarrhea, malabsorption, pernicious anemia, intestinal obstruction (due to fibrous stricture), peri-rectal disease (fistula, fissure, abscess) Ulcerative Colitis: Severe ulcerating continuous inflammation extending into the mucosa & submucosa, & limited to the colon & rectum ∙Morphologic Findings: Typically found in Caucasian, young, females. Involves the colon as a diffuse mucosal disease with distal predominance (typically the rectum). Full-thickness histological section will show disease limited to the mucosa ∙Clinical Features: Bloody/mucoid diarrhea, pain, fever, weight loss, relapsing episodes flaring up during emotional stress. Can lead to perforation, increased risk of colon cancer, and development of liver diseases (sclerosing cholangitis and bile duct carcinoma)

Caroli's Disease Define & Include Treatment

Cystic Dilatation of intrahepatic bile ducts which prevents the bile from floating freely and leads to recurrent cholangitis and cholelithiasis, and slow destruction of the liver Treatment: symptomatic until Liver transplant

Which lipase cleaves triglycerides to allow muscle and adipose fuel uptake? A. Adipose triglyceride lipase B. Hepatic triglyceride lipase C. Hormone sensitive Lipase D. Lipoprotein lipase E. Pancreatic lipase F. Phospholipase A2 G. Phospholipase C

D. Lipoprotein lipase

A 26-year-old female presents to your office with three day history of fever, chills, abdominal pain and bloody diarrhea. The patient recently returned home from a trip to Africa where she ate the local food and went swimming in a lake. You obtain stool microscopy which shows E. histolytica. Assuming she has no risk factors as above, what is the most appropriate treatment option for this patient? A. Metronidazole B. Paromomycin C. Iodoquinol D. Metronidazole plus Paromomycin E. Tinidazole

D. Metronidazole plus Paromomycin Invasive disease (fever)-- give the systemic (Metronidazole) + Luminal (Paromomycin)

A 68-year-old female with poorly controlled blood sugars secondary to long-standing diabetes presents to the ER for abdominal pain. An extensive workup including CT scan, ultrasound, EGD and colonoscopy are negative. A gastric emptying study is consistent with diabetic gastroparesis. Along with controlling the patient's blood glucose levels, she is started on metoclopramide. She develops a side effect and wishes to try a different agent for her gastroparesis. Which type of medication do you prescribe? A. 5-HT3 receptor agonist B. 5-HT4 receptor agonist C. Motilin receptor antagonist D. Motilin receptor agonist E. Acetylcholinesterase inhibitor

D. Motilin receptor agonist

A 23-year-old female HIV patient is losing weight. She states he has nausea and no appetite for the last 3 months. She is still sexually active and has no source of active infection. No new medications have been started. What should you do for this patient? A. Prescribe droperidol. B. Prescribe dronabinol. C. Prescribe promethazine. D. Prescribe a urine hcg test. E. Prescribe HAART therapy.

D. Prescribe a urine hcg test Sexually active, of child bearing age-- always do a pregnancy test

Gallstone Disease in Pregnancy

Decreased gallbladder contractility during pregnancy leads to increased risk of gallstone-- Laparoscopic surgery is safe to be performed after second semester

Gastric and Duodenal Volvulus

Defect in normal fixation of the stomach or duodenum which allows for twisting the occur Patient may present with chest pain, vomiting, or both; there is usually a triad of symptoms which include pain, unproductive retching, and an inability to pass a NG tube If symptoms warrant, surgical intervention is needed

Hartnup Disease

Defect in the transport of neutral amino acids resulting in a tryptophan deficiency that leads to Pellagra-like symptoms (photosensitive rash, ataxia) Patient must be administered a Niacin and high protein diet

Alcoholic liver disease Metabolism, epidemiology, genetics, & enzymes

Degenerative liver condition which occurs in (3) stages: 1. Fatty Liver 2. Alcoholic Hepatitis 3. Cirrhosis Ethanol is broken down by ADH (alcohol dehydrogenase) & CYP2E1 to Acetaldehyde (toxic- leads to flushing & nausea), which will be transformed into Acetate via ALDH (aldehyde dehydrogenase) In males, there is a greater level of ADH than in females, putting them at greater risk of alcoholic liver disease

Hepatitis D Lab Findings, Transmission, Pathogenesis, Disease Caused, Diagnosis, & Treatment

Delta Virus: circular RNA, enveloped virus Transmission: Blood & bodily fluids; epidemic in IV drug users Pathogenesis: Requires Active Hepatitis B to infect, because it steals the surface antigen from Hepatitis B in order to become infectious → This allows it to target cell proteins and replicate Disease: Increases severity of Hepatitis B infection ∙Fulminant infection is severe → alters brain function, extensive jaundice, & massive hepatic necrosis Diagnosis: HDAg, HDV RNA, Immunohistochemical staining of HDAg in liver, HDV IgM antibody, HDV IgG antibody confers immunity Treatment/ Prevention: No specific Treatment; treat same as Hepatitis B ∙ Immunization with Hepatitis B vaccine protects against Hepatitis D

A thirty-year-old female presents to her primary care provider complaining of diarrhea and fatigue for the previous few months. The patient's chart reveals that she has lost weight since her last visit and states that she hasn't changed her diet or physical activity. Celiac disease is suspected. How might this diagnosis be confirmed? Describe what is causing her symptoms.

Diagnosis Confirmation: Elevated serum IgA antibodies to gliadin, tissue transglutaminase (tTG), and endomysial antigens (EMAs), and genetic testing for HLA-DQ2 and HLA-DQ8. In addition, an intestinal biopsy would reveal atrophic villi and epithelial changes Cause of Symptoms: Immune system reacting to gliadin (a protein that is found in gluten), which causes the immune system to produce antibodies to gliadin, tTG, and EMAs, as well as attack the epithelial cells of the small intestine. As her small intestine has been damaged, she has reduced ability to digest (loss of brush border enzymes) and absorb (less surface area) dietary macronutrients. The reduced caloric availability translates into fatigue and weight loss. The reduced ability to absorb carbohydrates may contribute to the diarrhea.

Describe the digestion and absorption of: Carbohydrates Including location, as well as enzymes and transporters involved.

Digestion 1. Begins in the mouth, where salivary α-amylase will hydrolyze α-1,4 bonds in starch molecules 2. Salivary α-amylase is inactivated by low pH of the stomach 3. Digestion will continue in the small intestine, where brush border enzymes will cleave specific carbohydrates. Pancreatic α-amylase will hydrolyze α-1,4 bonds. β- Glucoamylase will hydrolyze the α-1,4 bonds of amylose, amylopectin, glycogen, and maltose. Sucrase will hydrolyze sucrose, maltose, & maltotriose. Isomaltase will hydrolyze the α-1,4 bonds in maltose and maltotriose. β-Glycosidase will cleave the β-1,4-bond in lactose. Trehalase will hydrolyze α-1,1 glycosidic bond in trehalose Absorption 1. Carbohydrates will be absorbed in the duodenum where transporters in the intestinal epithelial cells are located. 2. SGLUT1 will absorb Glucose & Galactose. 3. GLUT2 will absorb Glucose, Galactose, and Fructose 4. GLUT5 will absorb only Fructose 5. Facilitated glucose transporters will then transport the sugar from the intestinal cell into the serosa

Describe the digestion and absorption of: Proteins Including location, as well as enzymes and transporters involved.

Digestion 1. The low pH of the stomach will begin to denature proteins. Pepsinogen will activate pepsin at low pH, and pepsin will then cleave the peptide bonds on denatured proteins 2. Digestion continues in the small intestine where Pancreatic proteases (Trypsin, Chymotrypsin, Elastase, Carboxypeptidase A&B) will further degrade the protein by cleaving specific peptide bonds. Aminopeptidases on the bush border will further digest the protein into dipeptides, tripeptides, or single amino acids Absorption 1. Amino acid transporters (most of which are sodium dependent) will transport amino acids from the enterocyte into the blood 2. Small peptides can cross into the blood either by secondary active transport (with H+) or via endocytosis

Describe the digestion and absorption of: Fats Including location, as well as enzymes and transporters involved.

Digestion 1. There will be no significant digestion of fats until they reach the small intestine 2. In the small intestine, long chain fatty acids will be emulsified by bile salts (synthesized in the liver, stored in the gallbladder) increasing surface area. Colipase will move bile salts out of the way to allow Pancreatic Lipase access to digest triglycerides, cleaving them into (2) fatty acid chains (carbon 1 and 3) and (1) fatty acid + monoacylglycerol molecule 3. The (2) Fatty Acids and (1) fatty acid + monoacylglycerol will be surrounded by a micelle Absorption 1. Micelles will be absorbed by the intestinal epithelium where Fatty acids and the 2-monoacylglycerol will be re-synthesized into triacylglycerol 2. Lipoproteins (containing apoproteins- the protein part ApoB48 for chylomicrons and ApoB100 for VLDL) will provide a transport mechanism (Chylomicrons- ingested lipids & VLDL- endogenous lipids) for triglycerides since they are not water soluble. Triglycerides will be within the center of the transport molecule 3. Chylomicrons will start to enter the blood, 1-2 hours after a meal

Pre-Eclampsia & Eclampsia

Due to arteriolar constriction which causes vascular endothelial damage, platelet deposition & fibrin deposition Pre-Eclampsia: HTN with proteinuria and edema in 5-10% of pregnancies Eclampsia: Pre-eclampsia findings + seizure (possibly renal failure, coagulopathy, microangiopathic hemolytic anemia)

Which digestive protease becomes active through auto-catalysis? A. Aminopeptidases B. Carboxypeptidases C. Chymotrypsin D. Elastase E. Pepsin F. Trypsin

E. Pepsin

What do nascent chylomicrons pick up from HDL particles in order to mature? (select all that apply) A. apoA1 B. apoB48 C. apoB100 D. apoCI E. apoCII F. apoCIII G. apoE

E. apoCII G. apoE

What is the linkage between monosaccharides that compose lactose? A. α-1,1 B. α-1,4 C. α-1,6 D. α-1, β-2 E. β-1,4

E. β-1,4

How does Blood urea Nitrogen (BUN) level change in a patient with a bleed?

Elevated in upper GI bleeding due to breakdown of blood proteins to urea by intestinal bacteria and their subsequent absorption

Colonic diverticulum Epidemiology, Pathophysiology, and Presentation

Epidemiology: A pseudo-diverticulum/ pocket (not containing all layers of the GI) of the colon, almost always directly related to age (>40) and those with low fiber diets Pathophysiology: Thought to be caused by diminished fiber in diet, causing harder stool with greater sigmoid pressure required for evacuation or Irritable Bowel Syndrome Presentation: Pain, inflammation (diverticulitis), painless hemorrhage

inflammatory bowel disease (IBD) Include epidemiology, pathogenesis, and Extraintestinal manifestations

Epidemiology: Most frequently found in females in their teens and early 20's Pathogenesis: A chronic condition resulting from inappropriate mucosal immune activation. Results in inflammation of the lining of the GI tract, disrupting the body's ability to digest food, absorb nutrients, and eliminate waste; includes Crohn's disease and Ulcerative Colitis The primary symptom will be diarrhea >6 months which is often relapsing and remitting Extraintestinal manifestations: Polyarthritis, Uveitis, Sclerosing cholangitis (chronic fibrosing inflammation of biliary system), Sacroiliitis (inflammation of SI joint), Skin manifestation (erythema nodosum & pyoderma gangrenosum)

Adenocarcinoma of the colon: epidemiology, pathogenesis, morphologic findings, & clinical features

Epidemiology: Typically occurring in patients 60-70 years old Pathogenesis: APC gene mutations will lead to dysplastic epithelium which leads to increased formation of polyps. K-ras mutations then lead to formation of a polyp which forms an adenoma. Finally, a mutation in p-53 increases the expression of COX, allowing for progression to carcinoma Morphologic Findings: Protrusions into the lumen with ulceration and raised margins. On barium enema, may see "apple core" lesion- typically left sided tumor. Histology will show well to poorly differentiated, marked desmoplasia, mucin producing, "dirty necrosis" Clinical Features: Remains asymptomatic for years. Right colonic cancers will tend to cause bleeding, leading to fatigue, weakness, and iron deficiency anemia. Left colonic cancers cause abnormal bowel habits (diarrhea, constipations, melena) and often present with obstruction. Liver will be the primary site of metastasis

Enteroendocrine cells

Epithelial cells of the small intestine which secrete endocrine hormones into the blood stream (CCK, Secretin, GIP, and motilin) as well as paracrine hormones to act on the surrounding cells (somatostatin and histamine)

Gallstones

Excess in bilirubin in the blood forms stones which can block the bile duct, making bile salts unable to reach to small intestine Patient will be unable to digest fats leading to deficiencies in fat-soluble vitamins

Which lipase cleaves dietary lipid to produce lysophospholipid + fatty acid A. Adipose triglyceride lipase B. Hepatic triglyceride lipase C. Hormone sensitive lipase D. Lipoprotein lipase E. Pancreatic lipase F. Phospholipase A2 G. Phospholipase C

F. Phospholipase A2

Which digestive protease cleaves peptide bonds at the carboxy terminal to a lysine or arginine A. Aminopeptidases B. Carboxypeptidases C. Chymotrypsin D. Elastase E. Pepsin F. Trypsin

F. Trypsin

How can you determine if there is fibrosis vs. cirrhosis of the liver using non-invasive measures?

Fibrosis: Normal platelet count, elastography (assess stiffness of the liver) Cirrhosis: Decreased platelet counts (<125/cc), Increased AST, elastography (assess stiffness of the liver), serum creatinine >2.5mg/dL doubles 30 day mortality

Villi Histology

Finger-like projections of the small intestine mucosa that are formed by mucosa only and do not contain a submucosal core. The will contain arteries, veins, and lymph vessels (central location, known as a central lacteal)

Hepatitis C Lab Findings, Transmission, Pathogenesis, Disease Caused, Diagnosis, & Treatment/ Prevention

Flavivirus: ssRNA, (+) sense, enveloped Transmission: Blood & body secretions; major cause of post-transfusion hepatitis and IV drug users Pathogenesis: Gets in the blood where it coats itself with LDL or VLDL (lipids) in order to be taken into hepatocytes, and CD81 to be taken into WBC. Proteins will inhibit apoptosis preventing cell death and promoting persistent infection. Cell-mediated immune response is responsible for tissue damage Disease: non-A, non-B viral hepatitis (NANBH); Chronic Infection ∙ Acute: flu-like symptoms ∙ Severe/Chronic: progression to cirrhosis and liver cancer Diagnosis: HCV Ab, HCV RNA Treatment/ Prevention: ∙ "-previr" medications: protease inhibitors (Mavyret) ∙ RNA polymerase Inhibitors: Sofosbuvir will target (NS5B) & Ledipasvir will target (NS5A) preventing viral replication

Hepatic Lobule

Functional unit of the liver which is Hexagonal in shape and contains a central vein in the center Portal Triad will be on the periphery, where it contains the (i) Hepatic Artery, (ii) Portal Vein, (iii) Bile Duct, and (iv) lymph vessels

Hemochromatosis Pathogenesis, Symptoms, Biopsy findings, Lab findings, & Treatment

Genetic condition in which there is excess iron accumulation in the body (including the liver) due to a defect in HFE gene (C282Y, H63D) It results in chronic active hepatitis > Cirrhosis > cancer Pathogenesis: Accumulation of 3-5x more iron per day than a non-affected individual due to Hepcidin mutation (which is a negative regulator of iron absorption) Symptoms: ("Bronze Diabetes") Vertigo, Memory loss, Hair loss, heart degeneration, Hepatomegaly, Cirrhosis, DM, Testicular atrophy Biopsy: Iron accumulation in hepatocytes as noted by blue stain Labs: Elevated iron/ Transferrin saturation (>60% males, >50% females), Feritin >500, Genetic testing HFE gene mutation Treatment: Phlebotomy 1-2 units per week, then 1 unit every 3-4 months when appropriate levels (ferritin <50) are met, chelation agents, transplant

Cystic Fibrosis

Genetic disorder in which there is a defect in the cystic-fibrosis transmembrane receptor (responsible for pumping chloride) making it so secretions are not thinned (by water) and leading to blockage of digestive enzyme secretion and therefore undigested food Patients will present with stunted growth, foul smelling & bulky stool (oily) Patient must be treated with pancreatic enzymes

Child-Pugh Criteria

Grades level of liver disease

What are the clinical features of: Primary Biliary Cirrhosis

Granulomatous (autoimmune) destruction of bile ducts in portal triads, leading to increased retention of bile salts which can pruritis; patients also present with jaundice and painful hepatosplenomegaly

Coagulation factors

Group of plasma protein substances (Factor I-XIII), most of which are synthesized by the liver, contained in the plasma which act together to bring about blood coagulation

HBV Serology Panel

HBcAb (core antibody): first antibody to appear that indicated recent infection and can be used during "window period" → never found in vaccinated patient; IgM denotes acute or chronic infection HBsAg (surface antigen): present in any disease state; it is a marker of infection HBsAb (surface antibody): marker of protection/ immunity; indicates a previous infection or vaccinated state HBeAg (envelope antigen): correlated with active viral replication; present during acute disease and active chronic disease HBeAb (envelope antibody): present in chronic disease

Which Hepatitis viruses are transmitted via fecal-oral?

HEPA and HEPE "It must be something you AtE"

Which Hepatitis viruses are transmitted via blood and bodily fluids?

HEPB & HEPD "BlooD"

How is HEV is affected by pregnancy?

HEV can cause fulminant hepatitis in the 3rd trimester which increases the rate of death in pregnant women (25%)

What gene is associated with Hereditary Hemochromatosis?

HFE gene (C282Y, H63D)

Hepatitis B Lab Findings, Transmission, Structure, Pathogenesis, Disease Caused, Diagnosis requirements for chronic & resolved Hepatitis, & Treatment/ Prevention

Hepadna virus: the only DNA virus with a reverse transcriptase, enveloped Transmission: Blood & bodily fluids Structure: Surrounded by HB core antigen (HBcAg), with an envelope that contains HB surface antigen (HBsAg) as well as HB e antigen (HBeAg) ∙ Stable Virions (DANE = DNA + e): contains everything that it needs for replication Pathogenesis: HBsAg particles are released into the serum, which are not infectious but look like the virus, and binds all the antibodies. A neutralizing antibody (nAb) will be made against the HBsAg which can form immune complexes with HBsAg causing vasculitis, arthralgia, rash, & renal disease. Symptoms are caused by cell-mediated immune response Disease ∙ Acute: long incubation period resulting in flu-like symptoms, jaundice, icterus, dark urine, & pale stools ∙ Chronic: leads to cirrhosis & liver cancer, histology will have "grounded glass" appearance Diagnosis for Chronic: 1. HBsAg-Positive: 6 months 2. Serum HBV DNA present (>20,000) 3. Persistent or intermittent elevation of ALT/AST 4. Liver Biopsy shows chronic hepatitis Diagnosis for Resolved: 1. HBsAg-Negative 2. Previous history of chronic or acute HBV or presence of HBcAb 3. Undetectable serum HBV DNA 4. Normal ALT levels Treatment/Prevention: Prevention: HBV subunit vaccine (recombinant) given in a series of 3 injections ∙ In adults -- self limiting ∙ In Infants -- most become chronic Treatment: Used for chronic HBV, Protease Inhibitors (Entecavir or Tenofovir) or Pegylated interferon

Hepatitis E Lab findings, transmission, & disease caused

Hepevirus, RNA, naked Transmission: Fecal-Oral; typically contaminated water; endemic in Central & South Asia & India Disease: flu-like symptoms which only become serious in Pregnant women

Alkaline Phosphatase (ALP)

Hydrolase enzyme responsible for removal of a phosphate group from various molecules Present in the placenta, intestine, kidney, bone, and liver. When elevated, it suggests Cholestasis (reduction/ blockage of bile flow through the liver) when elevated It looks at flow through the bile duct It can also indicate tumors of the lung or ovary

Autoimmune Cholangitis Pathology, Symptoms, Lab findings, & Treatment

Identical to Primary Biliary Cirrhosis (PBC), except they are AMA Negative Presumably, and autoimmune disease of the liver which leads to progressive cholestasis and often end-stage liver disease Pathology: Immune destruction of small bile ducts in the liver (lymphocytes & plasma cells) Symptoms: Fatigue, Pruritus, RUQ discomfort, 25% are incidentally diagnosed, Osteopenia/ Osteoporosis, Fat soluble vitamin deficiencies, Steatorrhea Labs: Hypercholesterolemia, Elevated ALP, Elevated immunoglobulin levels (IgM), Elevated GGT, presence of ANA's, hallmark → AMA Negative Treatment: Ursodeoxycholic acid (UDCA)-- bile acid, bile acid sequestrants for pruritus

ischemic colitis Presentation and Treatment

Idiopathic, acute, self-limited compromise in intestinal blood supply, which is inadequate for meeting the metabolic demands of a region of the colon (typically splenic flexure or sigmoid colon) Patients typically present with abdominal pain and tenderness over the affected segment of bowel, followed by passage of blood mixed with stools Treatment: Supportive care with intravenous fluids, optimization of hemodynamic status, avoidance of vasoconstrictive drugs, bowel rest, and empiric antibiotics within 1 to 2 days

When should a prophylactic cholecystectomy be performed?

If there is a gallbladder polyp >1cm Avoid the chance of Gallbladder Adenocarcinoma

What is the immune response to Helminths?

IgE will bind to the Helminth in the tissue, Eosinophils recognize IgE and will release perforin and major basic protein to kill the worm

Hepatic Encephalopathy

Impaired ammonia metabolism, commonly due to cirrhosis, causes cerebral edema This leads to patient's who are delirious, disoriented, and confabulating-- patients may get lost & have day/night sleep reversal

Space of Disse

In the liver, it is the space between sinusoids and hepatocytes filled by collagen fibers and microvilli

Definitions and causes of constipation

Includes Straining, hard stools, & the inability to have a bowel movement (< 3 bowel movements/week); Type I and II on the Bristol stool chart Causes: 1. Dyssynergic Defecation: pelvic floor dysfunction typically seen in a female who has given birth or an episiotomy 2. Slow Transit Constipation: slow movement of stool through the colon 3. Normal-transit Constipation: normal movement of stool through the colon; typically caused by diet or medication

Review definitions and causes of diarrhea

Increase in daily stool weight above 200 grams with increased liquidity Causes 1. Secretory Diarrhea: cholera, toxin; does not stop with fasting 2. Osmotic Diarrhea: maldigestion, nutrients left in lumen; stops with fasting 3. Motility related: abnormal motility of GI tract (vagotomy, diabetic neuropathy) 4. Inflammatory Diarrhea: damage to the mucosal lining or brush border (bacterial infections, viral infections, parasitic infections, autoimmune problem)

Benzodiazepine as an Anti-emetic Indications, Side Effects, & Examples

Indications: Relatively weak anti-emetic agents, but can be useful in cancer patients Side Effects: Sedation & Respiratory depression Examples: Midazolam, Lorazepam

A 3-year-old boy presents with flatulence, bloating, and pain. His mother reveals that the symptoms become worse after drinking fruit juice. A hydrogen breath test reveals higher than normal levels of hydrogen after ingesting large levels of fructose. What is the most likely explanation for his symptoms and test results?

Inefficient absorption of the monosaccharide fructose. Rather than being absorbed through GLUT5 facilitated transporters, it is metabolized by gut bacteria. The metabolism of fructose by gut bacteria produces some waste gas products, including H2 that can be detected using a breath test.

Liver Fluke (Clonorchis sinensis) Infectious Form, Transmission, Disease Caused, & Treatment

Infectious Form: Cercariae Transmission: ∙ Water cress Disease: ∙ Upper Right quadrant pain, Fever, Diarrhea ∙ Hepatomegaly, Anorexia, jaundice ∙ Major cause of bile duct cancer (Cholangiocarcinoma) Treatment: ∙ Praziquantel

Schistosome Mansoni Infectious Form, Transmission, Diagnosis, Disease Caused, & Treatment

Infectious Form: Cercariae Transmission: Cercariae are released in water by snails, and when stepped on, penetrate the skin → lymphatics → blood → mesenteric vessels where the worm will grow and release eggs where they can be released into GI tract Diagnosis: ∙ Large, oval shaped egg with a spine on the side (mansoni) Disease: ∙ Bloody Diarrhea ∙ If adult worms infect portal vein, fibrotic tissue will deposit around the portal vein causing "Clay pipestem fibrosis" (portal hypertension) Treatment: ∙ Praziquantel

Giant Intestinal Fluke (Fasciolopsis buski) Infectious form, Transmission, At-risk, Diagnosis, Disease Caused, & Treatment

Infectious Form: Cercariae Transmission: Ingestion of contaminated water chestnuts → lays down over large intestine, causing malabsorption At-Risk: Immigrants & travelers of China, Vietnam, Thailand, India, & Malaysia Diagnosis: Large, Bile-Stained, Operculated (have hat) eggs in stool Disease: ∙ Malabsorption causes Steatorrhea (malodorous, greasy stool) ∙ Diarrhea alternating with constipation Treatment: ∙ Praziquantel

Giardia Lamblia Infectious Form, Transmission, Diagnosis, At-risk, Disease Caused, & Treatment

Infectious Form: Cyst- helps the Protozoa survive harsh environments Transmission: Beaver & Muskrat Reservoirs ∙ Fecal-Oral; drinking contaminated well water Diagnosis: ∙ "Pear or Teardrop shaped organism with bi-lobed nucleus" ∙ "Falling-leaf motility" At-Risk: ∙ Travelers (campers/hikers) ∙ Children in Daycare centers ∙ Institutionalized ∙ Homosexual men Disease: ∙ Watery Diarrhea (Steatorrhea): foul-smelling, which are mushy and greasy (Fat-rich Ghirardelli chocolates) Treatment: ∙ Metronidazole

Entamoeba Histolytica Infectious Form, Transmission, Diagnosis, At-risk, Disease Caused, & Treatment

Infectious Form: Cyst- helps the Protozoa survive harsh environments Transmission: Fecal-Oral (cysts passed in stool); cyst in water Diagnosis: ∙ Trophozoites with engulfed RBC or Cysts with multiple nuclei in stool "Entamoeba eats erythrocytes" At-Risk: ∙ Travelers to endemic area (camping/hiking) ∙ Immigrants ∙ Men who have sex with Men ∙ Immunocompromised patients ∙ Institutionalized (prison, daycare) Disease: ∙ Amebiasis/ Amebic Dysentery: Lytic effects on cells inducing apoptosis and bloody diarrhea ∙ Liver Abscess: creates Flask Shaped Ulcers Treatment ∙ Metronidazole ∙ Paromomycin

Cryptosporidium Parvum Infectious form, Transmission, Diagnosis, At-risk, Disease Caused, & Treatment

Infectious Form: Cyst- helps the Protozoa survive harsh environments Transmission: Fecal-oral: ingestion of contaminated food or water Diagnosis: ∙ Acid-Fast staining of oocytes in stool At-Risk: ∙ #1 cause of Watery Diarrhea in AIDS patients ∙ Animal Handlers ∙ Children Disease: ∙ Watery Diarrhea: scant, but multiple times Treatment: ∙ Fluid replacement ∙ Nitazoxanide for immunocompromised

Crystoisospora (Isospora) Belli Infectious Form, Transmission, Diagnosis, Disease Caused, & Treatment

Infectious Form: Cyst- helps the Protozoa survive harsh environments; will have Transmission: Fecal-Oral ingestion of mature oocytes which replicate inside the epithelium to cause tissue damage ∙ Oral/Anal Sexual Contact ∙ Contaminated food or water Diagnosis: ∙ Acid-Fast staining of oocytes in stool ∙ Oval-shaped cysts instead of round ones Disease: ∙ Steatorrhea in AIDS patients (mimic giardiasis) Treatment: ∙ TMP-SMX

Roundworm (Ascaris lumbricoides) Infectious Form, Transmission, Diagnosis, Disease Caused, & Treatment

Infectious Form: Eggs Transmission: ∙ Fecal-Oral: via food or water (eggs survive for months) ∙ Larval worm is ingested, enters the blood where it travels to the liver then lungs and is coughed up and swallowed Diagnosis: ∙ Eggs in stool are knobby-coated oval shaped, and bile stained Disease: ∙ Epigastric abdominal pain, Nausea, Vomiting ∙ Bolus can obstruct & perforate bowel ∙ Can migrate to liver ∙ When in lungs, can cause respiratory symptoms (mild asthma-like symptoms) Treatment: ∙ Bendazoles

Pinworm (Enterobius Vermicularis) Infectious Form, Transmission, Diagnosis, Disease Caused, & Treatment

Infectious Form: Eggs Transmission: ∙ Fecal-Oral: ingestion of eggs, which are "sticky" and can survive ∙ Autoinfection: transmission of egg from anus to mouth Diagnosis: ∙ Scotch-tape-test: clear tape applied to the anal area early in the morning before bathing, and it then examined via microscopy Disease: ∙ Nocturnal Perianal Pruritus: itching perianal area at night Treatment: ∙ Albendazole ∙ Pyrantel pamoate OTC -- for pregnancy

Diverticulitis History, Physical Examination, Complications, Lab Tests, & Treatment

Inflammation of colonic diverticula which can be associated with low Vitamin D levels History: Older patients, Lower abdominal pain, Change in bowel habits, hx of diverticulosis, Dysuria (pain/ burning with urination), Pneumaturia (air bubbles in urine) Physical Examination: Left lower quadrant abdominal pain often with palpable mass, Nausea with or without vomiting, Anorexia, Fever with elevated WBC Complications: Fistulization, Intra-abdominal abscess, Bowel Obstruction, Perforation with generalized peritonitis Laboratory: Elevated WBC, Elevated ESR Treatment: Bowel rest, Hydration, Broad spectrum antibiotics, in severe cases- hospitalization & surgical intervention (needle drainage procedures available)

If there is blood + PMNs in diarrhea, what kind of diarrhea is it?

Inflammatory invasive diarrhea

Peutz-Jeghers Syndrome

Inherited Autosomal Dominant disease, characterized by the association of GI polyposis and mucocutaneous pigmentation (dark blue/ brown macules around the mouth/ eyes/ nostrils/ perianal area). Polyps are typically found in the small intestine. Histologically, there will be a disorganized growth pattern with a branching network of connective tissue and smooth tissue, lined by normal intestinal epithelium Associated with STK11 Gene

Abetalipoproteinemia

Inherited inability to synthesize apoB48 (chylomicrons) and apoB100 (VLDL), thus lipids absorbed in the small intestine cannot be transported into the blood and therefore it builds up in enterocytes Leads to limited lipid absorption, caloric and vitamin (fat soluble) deficiencies, and steatorrhea

GI Bleeding will manifest as what laboratory result?

Iron Deficiency

Describe the etiologies and clinical subtypes of jaundice

Jaundice: yellowing of the skin and deep tissues as a result of bilirubin accumulation ∙ Overproduction ∙ Reduced liver uptake ∙ Impaired conjugation ∙ Decreased hepatic excretion ∙ Impaired bile flow Unconjugated (direct) Bilirubin: toxic to cells and insoluble at normal pH Conjugated (direct) Bilirubin: excess in plasma excreted by the kidneys, not toxic

Helicobacter Pylori Lab Tests, Transmission, Disease Caused, & Treatment

Lab Test ∙ Curved (spiral) Gram (-) Rod ∙ Catalase (+) ∙ Urease (+) ∙ Oxidase (+) ∙ Urea breath test used for diagnosis (looks for radiolabeled CO2) Transmission ∙ Fecal-Oral → urease produced ammonia + CO2 which creates an alkaline environment in which the bacteria can survive the acidic mucosa of the stomach Disease ∙ Peptic Ulcers ∙ Gastric Adenocarcinoma ∙ MALT B-cell Lymphoma Treatment ∙ Triple Therapy: Amoxicillin + Clarithromycin + PPI (Antibiotics Clear Pylori)

Bacillus Cereus Lab Tests, Transmission, Disease caused, & treatment

Lab Test ∙ Gram (+) Rod ∙ Aerobic Spore former Transmission ∙ Rice → spores survive cooking ∙ Contaminated meat, vegetables, sauce Disease ∙ Emetic Type → from contaminated rice; severe vomiting, nausea, and abdominal cramps → spores germinate ∙ Diarrheal Type → from contaminated meat/vegetables/sauces; severe watery diarrhea, nausea, and abdominal cramps → bacteria multiply in GI tract and release toxin Treatment ∙ Supportive Care

Clostridium Botulinum Lab Tests, Transmission, Disease Caused, & Treatment

Lab Test ∙ Gram (+) Rod ∙ Anaerobic ∙ Spore Forming ∙ Heat-labile Neurotoxin Transmission ∙ Intoxication (adults): heat-labile toxin inhibits ACh release at NMJ; typically from canned good ∙ Infection (children): ingestion of spores (honey) leads to disease state due to toxin produced in vivo Disease ∙ Food Borne Botulism (adults): weakness, dizziness, constipation, dry mouth, dilated pupils, death d/t respiratory paralysis ∙ Infant Botulism (children): flaccid paralysis, constipation Treatment ∙ Human Botulinum Immunoglobulin (Antitoxin) → blocks action of unbound toxin ∙ Gastric Lavage ∙ DO NOT use antibiotics

Clostridium Perfringens Lab Tests, Transmission, Disease Caused, Treatment

Lab Test ∙ Gram (+) Rod ∙ Anaerobic ∙ Spore-forming ∙ Heat Labile enterotoxin serotype A → reheating food degrades the toxin Transmission ∙ Ingestion of contaminated meat products Disease ∙ Diarrhea: watery, with abdominal cramps, no fever, no nausea, no vomiting Treatment ∙ Self-limited

Listeria Monocytogenes Lab Tests, Transmission, Disease Caused, & Treatment

Lab Test ∙ Gram (+) Rod ∙ Cold selection at 4°C ∙ CAMP test (+) ∙ Travels via Actin Jet Propulsion allowing intracellular movement and travel cell to cell ∙ Exhibits Tumbling motility Transmission ∙ Deli meat, soft cheeses ∙ Unpasteurized dairy products ∙ Cantaloupes Disease ∙ Fever, Myalgias, watery diarrhea Treatment ∙ Ampicillin

Clostridioides Difficile Lab tests, Transmission, Disease Caused, & Treatment

Lab Test ∙ Gram (+) Rod ∙ Spore-forming, Obligate anaerobic ∙ Production of Toxin A (enterotoxin that increases GI permeability) & Toxin B (cytotoxin that causes actin depolymerization, damaging the cell) Transmission ∙ Found in patients receiving broad spectrum antibiotics (most commonly clindamycin or ampicillin) that kill normal flora, but not C. Diff allowing it to replicate Disease ∙ Diarrhea → watery, green/yellow & very foul smelling ∙ Pseudomembranous Colitis → mucosa covered with white/gray/yellow patches containing fibrin, mucus, and leukocytes Treatment ∙ Stop broad-spectrum antibiotic ∙ Oral Vancomycin, Metronidazole, or Fidaxomicin ∙ Fecal Transplant

Staphylococcus Aureus Lab Tests, Transmission, Disease Caused, & Transmission

Lab Test ∙ Gram (+) cocci in clusters ∙ β-hemolytic ∙ Catalase (+) ∙ Coagulase (+) ∙ Heat stabile (100°C for 30 min) Transmission ∙ Intoxication → enterotoxins act as superantigens (i.e.- potato salad sitting out) ∙ Infected food handler Disease ∙ Ingestion → severe vomiting & watery diarrhea, NO fever within 2-6 hours Treatment ∙ Self-limited

Salmonella Lab Tests, Transmission, Disease caused, & Treatment

Lab Test ∙ Gram (-) Rods ∙ Lactose (-) ∙ H2S (+) ∙ Motile (have flagella) Transmission ∙ Improperly cooked poultry, eggs, dairy products ∙ Reptiles: turtles (snakes, lizards) ∙ Large number of organisms required ∙ S. typhi: Fecal-oral; person-to-person Disease ∙ Diarrhea: watery becoming bloody → elicits host-immune response that raises intracellular cAMP levels; no toxin present; nausea & vomiting ∙ Typhoid Fever: Salmonella typhi only; taken up by M-cell, and lives in it, circulating through the blood; fever, constipation, rose spots on torso, then progressing to diarrhea (pea-green which can be bloody) Treatment ∙ Salmonella non-typhi: Self-limiting ∙ Salmonella typhi: Ceftriaxone or Fluoroquinolone

Shigella Lab Tests, Transmission, Disease Caused, & Treatment

Lab Test ∙ Gram (-) Rods ∙ Oxidase (-) ∙ Lactose (-) ∙ H2S (-) ∙ Non-motile (no flagella) Transmission ∙ Fecal- Oral: seen a lot in daycares ∙ Travel via Actin jet propulsion which allows it to travel cell to cell; invades M-cells & can produce toxin ∙ Very few organisms will cause disease Disease ∙ Diarrhea: initially water, then develops into bloody stool (+PMNs) with abdominal cramping, & fever ∙ Bacillary Dysentery: diarrhea with neutrophils, RBC's, & mucus ∙ Hemolytic Uremic Syndrome: Shiga toxin causes anemia, thrombocytopenia, & acute kidney injury due to mechanical shearing of RBC Treatment ∙ Self-limiting ∙ Antibiotics to prevent spread: Fluoroquinolone (adult) & Ceftriaxone (children)

Campylobacter jejuni Lab test, Transmission, Disease caused, & Treatment

Lab Test ∙ Gram (-) curved rod ("comma shaped") ∙ Does not grow on MacConkey → required special campylobacter agar or Skirrow agar ∙ Oxidase (+) ∙ Grows at 42°C Transmission: ∙ Undercooked poultry ∙ Unpasteurized milk ∙ Person-to-person contact Disease ∙ Diarrhea: Grossly bloody + PMNs due to invasion of the colon ∙ Guillain-Barré syndrome: ascending symmetrical flaccid paralysis ∙ Reactive Arthritis: can't see, can't pee, can't climb a tree Treatment: Macrolides

Yersinia Enterocolitica Lab Test, Transmission, Disease caused, & Treatment

Lab Test ∙ Gram (-) pleomorphic rods ∙ Growth in 4°C Transmission ∙ Consumption of contaminated meat, milk, water ∙ Pet feces Disease ∙ Diarrhea: acute, bloody ∙ Pseudoappendicitis: RLQ pain, enlarged mesenteric lymph nodes Treatment ∙ Self-limiting

E. Coli Lab tests, Disease caused, Transmission, & treatment

Lab Test ∙ Gram (-) rods ∙ Lactose (+) → Grows on MacConkey ∙ Indole (+) Disease ∙ Enteropathogenic E. Coli (EPEC) → Pediatrics, watery diarrhea, vomiting; does not produce toxin, has a Type 3 secretion system (adheres to apical surface, flattens villi, prevents absorption) ∙ Enterotoxigenic E. Coli (ETEC) → Traveler's Diarrhea (Mexico), watery diarrhea, vomiting, cramps, nausea, low grade fever; produces heat-labile (↑cAMP), heat-stabile (↑cGMP) enterotoxin; there is no inflammation or invasion; can ferment sorbitol (SMAC Agar) and will produce pink colonies when plated ∙ Enterohemorrhagic E. Coli (EHEC) → Bloody diarrhea + no PMNs, cramps; no fever, Hemolytic uremic syndrome (shearing of RBC from shiga-like toxin), transmitted via undercooked meat (Hamburgers) & raw leafy vegetables or petting zoos; produces toxin; cannot ferment sorbitol (SMAC Agar) and will therefore be colorless when plated ∙ Enteroinvasive E. Coli (EIEC) → Fever, cramping, watery diarrhea; may have scant, bloody stools + PMNs; does not produce toxin, it is invasive and travels via actin jet trails which will allow it to move cell to cell; causes necrosis and inflammation of intestinal mucosa; can ferment sorbitol (SMAC Agar) and will produce pink colonies when plated Treatment ∙ NEVER treat EHEC with an antibiotic, it increases the risk of hemolytic uremic syndrome ∙ Most are self-limiting ∙ Supportive Care

Vibrio Cholerae Lab Tests, Transmission, Disease Caused, & Treatment

Lab Test ∙ Gram (-), curved Rods (comma shaped) ∙ Oxidase (+) ∙ Ferment Sucrose → Growth on TCBS agar (green agar turns orange) ∙ Flagellated ∙ Tolerate high salt concentrations (alkaline media) Transmission ∙ Growth in marine waters → associated with shellfish (raw shellfish) ∙ Fecal-oral transmission → Cholera toxin binds to and activates Gs proteins, increasing cAMP Disease ∙ Diarrhea: Profuse, "Rice Water" diarrhea, voluminous, colorless, and "almost" odorless Treatment ∙ Oral Rehydration

Lieberkühn's Crypts and Peyer's Patches

Lieberkühn's Crypts are simple tubular glands in the lamina propria of the small intestine, opening at the base of the villi. Between the glands, there will be cells of the immune system (lymphocytes, plasma cells, mast cells, macrophages, eosinophils). The lymphocytes often form lymphatic follicles (nodules) that aggregate in the ileum, forming Peyer's patches

Cameron's Lesion

Linear erosion in a hiatus hernia which can be chronic or acute Patient will not have abdominal pain, but may have reflux symptoms Patient should be given iron and a PPI

Pyogenic hepatic abscess

Liver abscess that can result from the spread of infection from inflammatory conditions such as appendicitis, diverticulitis, cholecystitis, cholangitis, and endocarditis On CT, they are usually hypodense with air and should be treated with antibiotics and percutaneous drainage

Intrahepatic Cholestasis of Pregnancy

Liver condition which is the second most common of jaundice in pregnancy which usually occurs in the 3rd trimester it will cause: Pruritus (palms and soles then spreading) and increased liver enzymes Delivery of the baby will resolve the symptoms in 24-48 hours. Prognosis for the patient is good, however, there is increased fetal meconium staining, stillbirth, and fetal distress

What is Triclabendazole used to treat?

Liver fluke due to Fasciola hepatica

Drug Induced Hepatotoxicity Cholestasis

Liver injury with Cholestasis due to medication (almost any → NSAIDs, PPI, H2RB, Antidepressants, Antibiotics, etc.) Seen with normal AST & ALT, and an increase in ALP → bile salts will accumulate in the tissue, causing the patient to become itchy (due to bile salt accumulation in the tissue) & jaundice Both symptoms almost always resolve when the drug is discontinued

Central Vomiting Center

Located in the Midbrain (medulla)

Pyrantel pamoate MOA, Indications, & Side Effects

MOA: Acts as a depolarizing neuromuscular blocking agent, causing sudden contraction followed by paralysis of the helminth-- Worm loses grip on intestinal wall and is passed out of intestine Indications: ∙ Pin Worm ∙ Roundworm Side Effects: ∙ Mild GI upset

Bulk forming laxatives MOA, Indications, Adverse Effects, Contraindications, and Examples

MOA: Adds mass to the stool in order to retain more water, while also forming an emollient gel that makes it easier for peristaltic action to move the stool Indications: Constipation, Diarrhea, Diverticulosis, Hemorrhoids, IBS, & maintenance of regular bowel movements Adverse Effects: Abdominal fullness, Bloating, Flatulence, Esophageal & colonic obstruction Contraindications: Caution in patients with esophageal strictures, ulcers, stenosis, intestinal adhesions, & phenylketonuria Examples: Methylcellulose (Citrucel), Polycarbophil (FiberCon), Psyllium (Metamucil)

Stimulant laxatives MOA, Indications, Adverse Effects, Contraindications, and Examples

MOA: Alters electrolyte transport and increases motor activity of the intestinal mucosa by directly irritating the smooth muscle (creating a contraction) and increasing net intestinal fluid accumulation Indications: Constipation Adverse Effects: Abdominal cramping, Hepatotoxicity, Discoloration of urine or feces (yellow/brown/pink/red/violet), Melanosis coli (discoloration of the colon due to chronic stimulant laxative use) Contraindications: Intestinal obstruction, acute intestinal inflammation (i.e.- Crohn's), appendicitis, pregnancy, abdominal pain of unknown origin Examples: Senna, Bisacodyl

Tetracycline (or mino/doxy) MOA, Indications, Side Effects, & Contraindications

MOA: Binds 30s and possibly 50s ribosomal subunit. Also causes alterations in the cytoplasmic membrane Indications: ∙ Balantidium coli Side Effects: ∙ Teeth Staining ∙ Photosensitivity ∙ Increased ICP Contraindication ∙ Pregnancy ∙ Very young children (< 8)

Bile Acid Sequestrants MOA, Indications, Adverse Effects, Contraindications, & Examples

MOA: Binds bile acids and some bacterial toxins Indications: Post cholecystectomy Diarrhea, C. Difficile associated diarrhea Adverse Effects: Heartburn, smells/ tastes bad Contraindications: DO NOT TAKE WITH MEDICATION -- it will bind the medication as well (take medication 2 hours prior to or after) Examples: Colesevalam (Welchol), Colestipol (Colestid), Cholestyramine (Prevalite)

Ivermectin MOA, Indications, Side Effects, & Contraindications

MOA: Binds chloride channel, chloride influx is enhanced and hyperpolarization occurs which results in paralysis Indications: ∙ Strongyloides (threadworm) (DOC) ∙ (Ancylostoma braziliense) Cutanous larvae migrans (DOC) Side Effects: ∙ Neurotoxicity (CNS depression, ataxia, MS changed) ∙ Mazzotti-type reaction: Fever, pruritus, lymph node enlargement and tenderness, hypotension, edema, abdominal pain, arthralgia Contraindications: Pregnancy

Trimethoprim-sulfamethoxazole (Bactrim) MOA, Indications, & Side Effects

MOA: Competitive inhibitor of dihydrofolate reductase (DHFR) inhibiting synthesis of tetrahydrofolate Indications: ∙ Cystoisospora belli (Isospora) ∙ Cyclospora cayetanensis Side Effects: ∙ Elevated serum creatinine (artificial, not AKI) ∙ Hyperkalemia ∙ Hypersensitivity vasculitis ∙ Hemolytic anemia with G6PD (DO NOT USE)

Osmotic laxatives MOA, Indications, Adverse Effects, Contraindications, and Examples

MOA: Composed of poorly-absorbed or non-absorbable sugars or ions which will cause intestinal water secretion, and increase stool frequency Indications: Colonoscopy Preparation & *Hepatic encephalopathy (Lactulose)* Adverse Effects: Dizziness (hypovolemia), Acute phosphate nephropathy, Abdominal cramping, gas, bloating Contraindications: *Chronic Kidney Disease (Magnesium Citrate)*, Patients with cardiac dysfunction, & patient's with Magnesium/Phosphorous/ Sodium restricted diets Examples: Magnesium Citrate, Sodium Phosphate, Lactulose, Polyethylene Glycol

Praziquantel MOA, Indications, Side Effects, & Contraindications

MOA: Increase permeability of trematode and cestode membrane to calcium resulting in paralysis, dislodgement and death Indications: ∙ Schistosomiasis (DOC) Side Effects: ∙ Headache, dizziness ∙ GI Effects Contraindications: Pregnancy, Ocular Cysticercosis

Niclosamide MOA, Indications, & Side Effects

MOA: Inhibits glucose uptake, oxidative phosphorylation, anaerobic metabolism in the tapeworm Indications: ∙ Fish Tapeworm Side Effects: ∙ GI symptoms ∙ Perianal Itching ∙ Unpleasant taste

Albendazole MOA, Indications, Side Effects, & Contraindications

MOA: Inhibits microtubule synthesis and glucose uptake of nematodes Indications: ∙ Taenia Solium- Neurocysticercosis (pictured) ∙ Roundworm ∙ Whipworm ∙ Pinworm Side Effects: ∙ Headache ∙ Alopecia ∙ Elevated LFT ∙ Acute Liver Failure ∙ Bone Marrow Suppression (monitor CBC) Contraindication ∙ Pregnancy ∙ Very young children (< 2)

Somatostatin Analog MOA, Indications, Examples

MOA: Inhibits secretion of gastrin, VIP, insulin, etc. (inhibition of GI tract) Indication: Secretory diarrhea (*hormone-secreting tumor*, chemotherapy induced, HIV associated, Diabetes associated) Examples: Octreotide (Sandostatin)

Metronidiazole (Flagyl) MOA, Indications, & Side Effects

MOA: Interacts with DNA to cause loss of helical DNA structure and strand breakage resulting in inhibition of protein synthesis and cell death Indications: Anaerobic & Parasitic infections (trophozoites, E. Histolytica) ∙ Entamoeba Histolytica ∙ Giardiasis ∙ Trichomoniasis Side Effects: ∙ Disulfram-like reaction: emesis with alcohol co-ingestion; do not use alcohol ∙ NOT used in pregnant females

Tinidazole MOA, Indications, & Side Effects

MOA: Interacts with DNA to cause loss of helical DNA structure and strand breakage resulting in inhibition of protein synthesis and cell death Indications: Anaerobic & Parasitic infections (trophozoites, E. Histolytica) ∙ Entamoeba Histolytica ∙ Giardiasis ∙ Trichomoniasis Side Effects: ∙ Disulfram-like reaction: emesis with alcohol co-ingestion; do not use alcohol ∙ NOT used in pregnant females

Paromomycin MOA, Indications, & Side Effects

MOA: Interacts with bacterial protein synthesis by binding to 16s ribosomal RNA of 30s ribosome subunits Indications: Luminal infections ∙ Intestinal E. Histolytica Side Effects: ∙ GI symptoms ∙ Bacterial/ fungal Superinfection (i.e.-C. Diff)

Nitazoxanide (Alinia) MOA, Indications, & Side Effects

MOA: Interference with pyruvate: ferredoxin oxidoreductase (PFOR), which is essential to anaerobic metabolism Indications: ∙ Giardiasis ∙ Cryptosporidium Side Effects: ∙ Headache ∙ Abdominal pain/ Nausea ∙ Yellowing of sclera and red/orange urine

Stool Softeners MOA, Indications, Adverse Effects, Contraindications, and Examples

MOA: Reduced surface tension of oil-water interface of the stool resulting in enhanced incorporation of water and fat, allowing for stool softening Indications: Constipation Adverse Effects: Intestinal obstruction, diarrhea, abdominal cramping Contraindications: Intestinal obstruction Examples: Docusate

Ondansetron (Zofran) MOA, Indication, Adverse Effects

MOA: Serotonin Antagonist (5-HT3 receptor) Indication: Chemotherapy and radiation induced emesis, post-operative emesis, hyperemesis of pregnancy, NOT USEFUL for motion sickness Adverse Effects: Headache, Abdominal pain, generally very well tolerated

Describe malabsorption & diarrhea. What associated disorders cause them?

Malabsorption: Defective absorption of fats, fat and water-soluble vitamins, proteins, carbohydrates, electrolytes and mineral, and water Causes- inadequate digestion, Inadequate absorption surface, Lymphatic obstruction, Primary mucosal disease Diarrhea: increase in stool mass, frequency, or fluidity, typically >200g/day Causes- secretory diarrhea, osmotic diarrhea (lactase deficiency), malabsorption, exudative (inflammation)

What is the space of Disse?

Marked by fenestrated endothelial cells, it is an area which RBCs cannot cross but that allows micronutrient exchange

Prokinetic Agents

Medications that enhance coordinated GI motility

Gamma Glutamyltransferase (GGT)

Membrane-bound peptidase that hydrolyzes peptides to amino acids and smaller peptides, which is particularly sensitive to alcohol consumption Found in the proximal renal tubule, liver, pancreas and intestine. GGT activity in serum comes primarily from liver Elevation can indicate cirrhosis, hepatitis, acute pancreatitis, acute cholecystitis, or nephrosis, and to test for Helicobacter pylori antibodies Increases with liver injury and cholestasis

Polycystic Liver Disease

Multiple cystic lesions on the liver which are often associated with polycystic kidney disease They rarely cause liver failure, but symptoms are related to compression and mass effect Therapy is resection

Tumors of the appendix

Neuroendocrine carcinoid tumors are the most common tumor of the appendix, Histologically they will stain with chromogranin and synaptophysin. Clinically, it is indistinguishable from acute appendicitis

What immune cells respond to a protozoan infection?

Neutrophils & Macrophages

Aorto-Enteric Fistula

Occurs in patients who have a history of abdominal aortic surgery, and presents with acute hematemesis. Patients develop a fistula between the aorta and duodenum, which bleeds initially then clots, but can DIC and cause death within hours.

Acute Fatty Liver of Pregnancy

Occurs in the 3rd trimester of pregnancy and begins with nausea, fatigue, malaise, & vomiting but then progresses to Jaundice, encephalopathy, and bleeding (DIC is common). There is >50% mortality On biopsy, will note microvesicular fat and macrovesicular fat Treatment: Delivery

Regenerating Nodule

Occurs in the presence of established cirrhosis and is related to excessive, uncontrolled proliferation of hepatocytes Should be observed and a biopsy taken

What is the diagnostic approach regarding Hepatitis B virus antigen and antibody testing

Only Viral Hepatitis which is DNA rather than RNA Serum IgM antibody is present in the acute phase Serum IgG antibody represents life long immunity Surface antigen (HBsAg) presents with active infection, while the antibody (HBsAb) does not rise until acute disease is resolved NOTE: if core (HBcAg) is present, this means the patient had a previous infection (this is not present in the immunization)

Extraluminal Duodenal Diverticulum

Outpouching in the duodenum which increase in frequency with age and are generally asymptomatic but can cause diverticulitis Since they are usually close to the ampulla of vater, they can increase incidence of gallstone and common bile duct stone

Intraluminal DIverticula

Outpouching that goes inward towards the lumen instead of outwards It is a congenital lesion that results from a failure of normal canalization of the duodenum; typically presents with duodenal obstruction

What are the enzymes used in drug metabolism and the role of each?

PHASE I ENZYMES- CYTOCHROME P450 → Reactions generate toxic compounds PHASE II ENZYMES-UDP GLUCORONYL TRANSFERASE, GLUCORONIDATION, SULFATION, GLUTATHIONE METABOLISM → Detoxification of toxic compounds made by the body by converting them to a water soluble, non-toxic product

Mallory Weiss Tears

Painless upper GI bleeding due to linear mucosal tear(s) near the Esophageal-gastric junction usually on the gastric side

Irritable bowel syndrome (IBS) Include pathogenesis & clinical features

Pathogenesis: A group of functional bowel disorders characterised by chronic, relapsing abdominal pain, bloating, and change in bowel habits without a structural or biochemical cause. Gross and microscopic evaluation of the bowel will be normal. Patients will c/o abdominal pain and bloating with a change in bowel habits that improves with defecation Diagnosed by exclusion, stress and hormonal fluctuations (menstrual cycle, pregnancy, thyroid disease) may be precipitating factors. Increased dietary fiber may improve symptoms Clinical Features: Typically seen in middle aged females, they commonly are anxious with symptoms aggravated by stress, bloating, abdominal pain, and relieved by defecation; mucus commonly in stool

Ischemic bowel disease Include: pathogenesis & clinical features

Pathogenesis: Acute occlusion that can affect one of the 3 major blood supplies (Celiac, SMA, IMA) and most often occurs in the "watershed zones" of the intestine (i.e.- splenic flexure, rectosigmal junction). Occlusion can be due to thrombosis/ embolism or atherosclerosis (occlusive type), or be caused by hypovolemia, dehydration, hypotension, etc. which leads to decreased cardiac output (non-occlusive type) Clinical Features: Typically presents with sudden onset of cramping in the LLQ, a desire to defecate, and the passage of blood or bloody diarrhea. Requires urgent surgery and visceral revascularization, followed by evaluation of viability of the bowel segments

What are the pathological & clinical features of: Alpha-1 Antitrypsin Deficiency

Pathological: Accumulated AAT appears as inclusions in hepatocytes that stain positive with PAS (periodic acid- Schiff) Clinical: AAT (inhibitor or proteolytic enzyme elastase) accumulates causing liver disease and cirrhosis (most common in children)

What are the pathological, clinical, and laboratory features of: Alcoholic Hepatitis

Pathological: Hepatic Steatosis Clinical: Fatigue, anorexia, nausea, RUQ pain, fever Laboratory: AST>ALT (2:1), rarely exceeding 500 IU/L; Maddrey's Discriminant Function >32 or MELD >18

What are the pathological, clinical, laboratory features, & treatment of: Non-Alcoholic Fatty Liver Disease

Pathological: Hepatocyte ballooning Clinical: Most common liver disease in US, it is generally found incidentally on routine laboratory testing with increase in liver enzymes; thought to cause a fatty acid d/t insulin resistance and obesity which increases triglycerides deposited in the liver -- can progress into Non-Alcoholic steatohepatitis (NASH) Laboratory: AST : ALT (1:1) Treatment: Weight loss, Pioglitazone (can treat steatohepatitis), Vitamin E (improves in non-diabetic adults, but increases overall mortality in diabetics)

What are the pathological, clinical, and laboratory features of: Autoimmune Hepatitis

Pathological: Hyperproliferation rosettes Clinical: fever/ jaundice/ hepatosplenomegaly Laboratory: (+) ANA and (+) ASMA (greater in adults than children)

Diverticular Bleeding

Penetration of a vessel through a diverticula causing bleeding, most common in the right colon Risk factors include: NSAIDs, Advanced Age Clinical Presentation: Sudden onset of maroon/bright red bleed, signs of significant blood loss, with no abdominal pain or tenderness ("painless hematochezia") Most will stop spontaneously

Hepatitis A Lab Findings, Pathogenesis, Transmission, Disease Caused, Diagnosis, Treatment/ Prevention

Picornavirus: (-) sense RNA, naked, Virus Pathogenesis: Virus is ingested, enters the bloodstream via the GI tract and replicates in hepatocytes & Kupffer cells then is released into the bile & stool; symptoms are due to cell-mediated immune response Transmission: Fecal-Oral; most often associated with an infected food handler or shellfish (clams, oysters, mussels) Disease: Causes Acute Hepatitis → Individuals are highly infectious before they exhibit any symptoms ∙ Icterus (yellow eyes), flu-like symptoms (fever/fatigue/nausea/abdominal pain) ∙ Fulminant disease can result in arthritis & rash Diagnosis: HAV IgM (+) during acute disease, HAV IgG (+) following resolved disease or vaccination Prevention: Killed vaccine (infants @ 1 yr + 2nd dose 6months later)

Compare & contrast the different types of colonic polyps including: A. Hyperplastic polyps B. Inflammatory polyps C. Hamartomatous polyps D. Juvenile polyps

Polyps are any elevation on the intestinal surface mucosa. Pedunculated polyps are attached to the surface via a narrow, elongated stalk, while Sessile polyps will have a broad base without a clear stalk A. Non-neoplastic hyperplasia of the colonic epithelium that leads to the formation of elongated glands with a sawtooth appearance on cross section. They are the most common type of polyp and are benign. Occur in the recto-sigmoid region during adulthood (6th and 7th decades) B. Results from chronic cycles of injury and healing (i.e.- IBD, Chronic Ulcerative Colitis), with patients often presenting with rectal bleeding and mucus discharge. They are benign with no cancer potential C. Benign tumor-like malformation made up of an abnormal mixture of cells & tissues found in areas of the body where growth occurs. Histologically you will see erosion and cystically dilated crypts filled with mucus, neutrophils, and debris D. A type of Hamartomatous polyp which is typically found in the rectum of children <10. They are Pedunculated, round, and have a smooth surface. Histologically, they are composed of mucus-filled cystic glands, are edematous, and have inflamed stroma. Patients c/o rectal bleeding, diarrhea, constipation, cramping, anemia, and chronic weight loss. Can increase risk of GI tumors

Identify the pathways of blood and bile flow through the liver

Portal Vein: poorly oxygenated blood from the gut (most of the blood flow) Hepatic Artery: oxygenated blood from the celiac artery Porta Hepatis: Entry of the portal vein and hepatic artery

Norovirus Define, MOA, Transmission, and Symptoms

Positive sense, naked RNA virus that will cause norwalk virus, Calicivirus, and Astrovirus. Will cause symptoms in both adults and children Compromises the intestinal brush border, preventing the absorption of water and nutrients → self-limiting Transmission: Fecal-Oral; commonly found on cruise-ships Symptoms: Vomiting, Watery Diarrhea, Fever, & Dehydration

Primary Biliary Cirrhosis (PBC) Pathology, Symptoms, Lab findings, & Treatment

Presumably, and autoimmune disease of the liver which leads to progressive cholestasis and often end-stage liver disease Pathology: Immune destruction of small bile ducts in the liver (lymphocytes & plasma cells) Symptoms: Fatigue, Pruritus, RUQ discomfort, 25% are incidentally diagnosed, Osteopenia/ Osteoporosis, Fat soluble vitamin deficiencies, Steatorrhea Labs: Hypercholesterolemia, Elevated ALP, Elevated immunoglobulin levels (IgM), Elevated GGT, presence of ANA's, hallmark → presence of AMAs (Antimitochondrial antibodies) Treatment: Ursodeoxycholic acid (UDCA)-- bile acid, bile acid sequestrants for pruritis

Explain the toxicity of acetaminophen, and who is at increased risk of hepatotoxicity

Primarily undergoes sulfonation & glucuronidation which inactivates the medication and make the metabolite water soluble, leading to elimination. A small amount of Acetaminophen will go through the P450 system forming a toxic compound which will react with Glutathione to create a non-toxic compound Ethanol, anti-seizure medications, & rifampin and increases the risk of acetaminophen hepatotoxicity because they increase the level of P450 so much more metabolism of Acetaminophen will occur through this system, creating a larger amount of toxic substance produced

Albumin

Protein made by the liver which is responsible for binding water, cations, fatty acids, hormones, bilirubin etc. in order to regulate oncotic pressure of the blood

What is the value of medical therapy of bleeding peptic ulcers

Proton Pump Inhibitors (PPI): PPI can help prevent recurrence because it will irreversibly inhibit the H+/K+ ATPase pumps in parietal cells, decreasing acid release, and therefore increasing pH to allow the clot to stabilize

What is the role of endoscopic therapy for control of GI Bleeding

Provides ability to suction the stomach and clean up the GI bleed as well as thermal (heater probe)/non-thermal (Epinephrine injection)/and mechanical (clips or hemospray to stop the bleed) measures to stop the bleed Early endoscopy (within 12-24 hours) is recommended for most patients with acute upper GI bleed and reduces death and decreases rate of hospitalization

Rotavirus Define, MOA, Transmission, Symptoms, & Prevention

Segmented, naked, dsRNA virus that is the most common global cause of infantile gastroenteritis → Major cause of acute diarrhea in the US during the winter especially in day cares and kindergarten → Adults DO NOT get this virus Causes Villous destruction with atrophy leading to ↓ absorption of Na+ and loss of K+; NSP4 protein promotes Ca2+ influx into enterocytes leading to secretion of water and loss of ions Transmission: Fecal-Oral Symptoms: Vomiting, Watery Diarrhea, Fever, & Dehydration Prevention: Live, attenuated oral vaccine

Understand serum bilirubin and causes of hyperbilirubinemia

Serum Bilirubin: a waste product that comes from the break down of RBCs, which will be bound to albumin, brought to the liver to become conjugated and then secreted through the common bile duct Hyperbilirubinemia: Excess bilirubin due to a number of etiologies → Conjugated: hepatocellular dysfunction, cholestasis, genetic disorder (Dubin-Johnson, Rotor) → Unconjugated: Hemolysis, reabsorption of hematoma, genetic disorders (Gilberts, Crigler-Najjar I/II) Patients will be jaundice

How does Hematocrit change during a bleed?

Single value may not reflect the degree of bleeding (it is representing the blood components to the volume), due to the fact that you do not bleed components, but volume -- typically doesn't catch up until 24-72 hours

Acetaminophen Toxicity Stages & management of overdose

Stage I: Occurs within several hours of ingestion and is characterized by anorexia, nausea and vomiting lasting up to 24 hours; occasionally, no GI symptoms occur Stage II: Relative feeling of well-being that persists for 48 hours after ingestion regardless of therapy. During this period, aminotransferase levels, bilirubin, and PT begin to rise, and the patient may begin to develop RUQ pain Stage III: Overt Hepatic Injury-AST levels may peak at greater than 20,000 u/l. Manifestations include Vomiting, Hypoglycemia, acidosis, bruising, bleeding, and encephalopathy including hepatic coma OVERDOSE: When Acetaminophen is broken down by the P450 system, it is broken down into a toxic product which Glutathione will change into a non-toxic product. In overdose, the body does not have enough Glutathione. N-Acetylcysteine (NAC): Replenishes hepatic glutathione; the sooner it is started, the less likely that liver failure will develop

What cells are involved in fibrosis?

Stellate/ Ito cells This can lead to cirrhosis

Cowden's Syndrome

Syndrome associated with Harmartomas in all sections of the bowel, linked to mutation of PTEN (which is a tumor supressor) VERY high risk of THYROID and BREAST cancer

Describe the physiological anatomy of the liver

The Liver Lobule is the functional unit of the liver. It is will contain the Bile duct (lined by cholangiocyte), Hepatic artery, Portal vein (all three which form the portal triad), & the lymphatic duct. The liver lobule will be made up of Hepatocytes as well as Kupffer cells (macrophages). The Space of Disse (below the sinusoid endothelium & hepatocytes) will allow movement of substances in and out of the hepatocyte. Stellate (Ito) cells will store Vitamin A and secrete growth factors (which allows the liver to regenerate)

Portal Hypertension

The elevation of blood pressure within the portal venous system Signs/Symptoms: Ascites, Jaundice, Development of collateral vessels (eosphageal, gastric) can lead to GI bleeds d/t how weak they are, vomiting, diarrhea Development: Cirrhosis will decrease hepatic blood flow (increases resistance)→ splanchnic vasodilation→ sends signal that BP is low→ Activation of vasoconstrictors→ Ascites

Discuss how the liver functions as a blood reservoir

The liver can hold 1300-1400mL of blood. If the pressure is too high (portal hypertension), however, the liver will start "leaking"

Meckel's Diverticulum

The most frequent congenital anomaly of the intestinal tract, it is an outpouching of the distal ileum Follows the rules of two ∙ Diagnosed around age 2 ∙ 2 ft from the ileocecal valve ∙ 2% of the population ∙ 2 tissue types (gastric and pancreatic) ∙ 2x more likely in males Most commonly asymptomatic but can lead to Hemorrhage, Perforation, Intussusception

An 18-month-old female is presented for a standard check-up and is found to be very small for her age. The mother reveals that the child's stools are very bulky and smell bad, and the soiled cloth diapers extremely difficult to clean. The toddler has dry skin and some hepatosplenomegaly, as well as a dry cough. Suspected diagnosis? Treatment?

The observations that the child is small for age (suggestive of insufficient nutrition), and bulky and smelly stools (food not being absorbed) that lead to difficult to clean soiled cloth diapers (suggestive of fat in stool point to malabsorption. Diagnosis: Cystic fibrosis-- A positive genetic test or positive sweat chloride test would help to confirm this diagnosis Treatment: Enterocoated digestive enzymes (not active until they reach the small intestine) taken with meals to allow for digestion of the macronutrients in the diet. The dosage will need to be adjusted as the child grows.

A 47-year-old male presents with sudden onset of severe epigastric pain that has become more severe over the last 16 hours. He c/o nausea and lack of appetite. He is tachycardic, and demonstrates abdominal tenderness and muscular guarding, with diminished bowel sounds. Laboratory tests, including elevated serum amylase and lipase, confirm acute pancreatitis. How will digestion of each of the following be affected? A. Carbohydrates B. Lipids C. Proteins

The presence of pancreatic amylase and lipase in the serum indicates that these enzymes were inappropriately released into the blood. A. Digestion of starches in the small intestine will be reduced due to insufficient pancreatic amylase secretion, although the brush border enzymes should still be able to cleave their carbohydrate substrates. B. Digestion of triglycerides will be greatly impacted due to insufficient pancreatic lipase secretion into the small intestine. C. Secretion of pancreatic proteases is also predicted to be affected during acute pancreatitis, preventing digestion of proteins in the small intestine. Digestion of all of the macromolecules will be impacted, with triglyceride and protein digestion most affected

Identify potential risk factors of prescribing thickened liquids and other food texture restrictions.

They are not well liked, there is a high refusal rate to drink them, & they become thicker as they sit Dehydration may develop due to person refusing to drink, leading to additional medical problems Typically prescribed because they move slowly, allowing for premature spillage and delayed airway protection without aspiration Includes: Nectar, Honey, & Pudding

What is the role of steroid in drug induced liver injury?

They have NO ROLE and should not be given; they can increase the risk of infection and negatively impact the ability to undergo liver transplant

What laboratory test finding is indicative of cirrhosis?

Thrombocytopenia/Leucopenia (Platelets < 130) → Hypersplenism leads to thrombocytopenia

If there is blood + no PMNs in diarrhea, what kind of diarrhea is it?

Toxin-mediated bacteria (EHEC)

Physiology of Emesis

Toxins (chemoreceptor trigger zone), smells/site/thought (cerebral cortex), motion (vestibular apparatus), and chemo/radiation therapy or gastroenteritis (pharynx/GI tract) will feed into the central vomiting center (medulla) and induce vomiting Understanding where the input is coming from, changes what medication you should choose

Whipworm (Trichuris trichiura) Transmission, Diagnosis, Disease caused, & treatment

Transmission: ∙ Fecal- Oral: ingestion of egg, which hatches and stays in the GI tract where they will lay more eggs Diagnosis: ∙ Barrel/ Football shaped egg with plugged ends in stool Disease: ∙ Abdominal Pain/ vomiting/ anorexia ∙ Bloody Diarrhea ∙ Rectal Prolapse ∙ Trichuris Dysentery Syndrome: bloody, mucoid Diarrhea, small frequent stools, anemia, & growth retardation Treatment: ∙ Bendazoles

Trichinosis (Trichinella spiralis) Transmission, Disease Caused, & Treatment

Transmission: ∙ Larvae ingested in undercooked meat (pork, bear, fox, walrus) will enter the blood stream and encysts in striated muscle Disease: ∙ Trichinosis: fever, vomiting, nausea, periorbital edema, myalgia Treatment: ∙ Bendazoles ∙ Bed rest, antipyretics, analgesics

Echinococcus Transmission, Diagnosis, Disease Caused, & Treatment

Transmission: ∙ Tapeworm of carnivores (dogs, foxes) in sheep raising areas ∙ Ingestion of eggs in foods contaminated with dog feces Diagnosis: ∙ CBC (Eosinophilia), ELISA, Imagine (X-rays for cysts) Disease: ∙ Hydatid cyst disease: cysts develop on liver & lungs which causes abdominal pain, hepatic mass, bile duct obstruction, chest pain, cough, hemoptysis; if cyst ruptures it can cause anaphylaxis Treatment: ∙ Surgical removal of cysts ∙ Albendazole/ Mebendazole

Hookworm (Ancyclostoma spp) Transmission, Disease Caused, & Treatment

Transmission: Eggs in the environment will hatch, when stepped on, they burrow in the skin and enter the lymphatics → blood → heart → lungs → it will be coughed up and swallowed Disease: ∙ Skin irritation → Respiratory Symptoms (mild asthma-like) → GI Symptoms (N/V/D) ∙ Iron Deficiency Anemia (microcytic anemia): due to sucking blood from intestinal wall ∙ Cutaneous Larva Migrans: pruritic, serpiginous rash from walking barefoot ∙ Loeffler Syndrome: pulmonary infiltrates with peripheral eosinophilia Treatment: ∙ Albendazole ∙ Iron Therapy for Anemia

Threadworm (Strongyloides stercoralis) Transmission, At-risk, Disease Caused, & Treatment

Transmission: Eggs in the environment will hatch, when stepped on, they burrow in the skin and enter the lymphatics → blood → heart → lungs → it will be coughed up and swallowed Eggs which are in the intestine can hatch there (unlike hookworm, which cannot) At-Risk: ∙ Immunocompromised Patients ∙ Transplant patients Disease: ∙ Vomiting/Diarrhea/Malabsorption ∙ Hyperinfection Syndrome: larvae spread throughout the body causing profound diarrhea, malabsorption, electrolyte imbalance, meningitis, & secondary bacteremia Treatment: ∙ Ivermectin

Toxocariasis Canis Transmission, Disease Caused, & Treatment

Transmission: Fecal-Oral ∙ Roundworms found in dog & cats → Humans are accidental hosts Disease: ∙ Visceral Larva Migrans: Eggs are ingested, larvae hatch and get "lost" --migrate to blood through the intestinal wall leading to inflammation which affects the liver, eyes (visual impairment/ blindness), CNS (seizures, coma), & heart (myocarditis) Treatment: ∙ Bendazoles

Anisakiasis Transmission, Disease Caused, & Treatment

Transmission: Ingestion of raw fish (sushi) Disease: occurs within hours after ingestion ∙ Violent abdominal pain, Nausea, Vomiting ∙ May mimic Crohn's disease Treatment: ∙ Surgery to remove larvae

Fish Tapeworm (Diphyllobothrium latum) Transmission, Diagnosis, Disease Caused, & Treatment

Transmission: Ingestion of undercooked freshwater fish ("Gefilte fish") Diagnosis: ∙ Operculated (has hat) egg in stool Disease: ∙ Megaloblastic Anemia: depleted serum B12 d/t tapeworm competition ∙ Epigastric pain, abdominal cramps, nausea, vomiting, weight loss Treatment: ∙ Praziquantel

Pork Tapeworm (Taenia Solium) Transmission, Diagnosis, Disease Caused, & Treatment

Transmission: Ingestion of undercooked pork (pig eats egg, egg hatches, and the worm burrows in muscle) Diagnosis: ∙ Proglottids (Segmented body) in feces ∙ Scolex (drill like head) Disease: ∙ Abdominal discomfort, diarrhea, indigestion ∙ Cysticercosis: ingestion of the egg which hatches in the body, and leads to calcification in the body → #1 Cause of seizures world-wide Treatment: ∙ Praziquantel

Colonic Volvulus Include Physical exam findings and treatment

Twisting of the colon on itself, most often occurring in the sigmoid colon or cecum which often occurs in older individuals, poor peritoneal fixation, and diets high in fiber without enough fluid intake Physical Exam: Abdominal distention, Abdominal series shows massive distention with a single loop of bowel shaped like a "coffee bean" Treatment: Barium Enema, Surgical

Liver Lesion: Simple Cyst

Typically asymptomatic, discrete/round/fluid density which very rarely require treatment unless they are symptomatic

Describe the clinical importance of measuring bile levels

Unconjugated bilirubin is found in the plasma or intestine, therefore if these levels are high, it is associated with a vascular issue or a conjugation problem ∙ Hemolytic disease ∙ Drugs inhibiting uptake into liver Conjugated bilirubin is found in hepatocyte, therefore if these levels are high, there may be a blockage

Goblet Cells

Unicellular mucin-secreting glands found in the villi of the small intestine with few microvilli. Mucinogen granules at the apex of the cell will appear empty

What is Bithionol used to treat?

Used for Fasciola hepatica (liver fluke)

What is Fumagillin used to treat?

Used in Enterocytozoon bieneusi infections (Microsporidiosis)

Review the CAGE questionnaire

Used to diagnose chronic alcoholism 1. Have you felt you should Cut down on your drinking? 2. Have people Annoyed you by criticizing your drinking? 3. Have you ever felt Guilty about your drinking? 4. Have you had an Eye-opener drink (first thing in the morning) to steady your nerves or get rid of a hangover? An answer of "yes" to two or more questions is positive and patient is at increased risk of developing alcoholic liver disease

Glucocorticoids for nausea

Usually used in combination with other medications for chemotherapy-induced emesis Dexamethasone is the drug of choice

What is used to confirm Hepatitis C Virus?

Viral Load HCV RNA is detectable in 1-3 weeks

Describe the toxicities associated with ingestion of excess vitamin A and vitamin D.

Vitamin-A: Excessive intake (>7.5 mg retinol/day) causes Hypervitaminosis A which results in dry skin, enlarged liver (which can become cirrhotic), & CNS symptoms which can mimic a brain tumor Vitamin-D: The most toxic vitamin, enhanced Ca2+ absorption and bone resorption result in hypercalcemia, leading to deposition of calcium in arteries & kidneys

What are the Vitamin K-dependent coagulation factors? What disease process is indicated by: A. Increased Prothrombin Time with Vitamin K dependent coagulation factors at a normal level B. Increased Prothrombin Time with Vitamin K dependent coagulation factors at a decreased level

Vitamin-K dependent coagulation factors: II, VII, IX, X A. Cirrhosis B. Prolonged cholestasis, Obstructive jaundice, Malabsorption, Fat malabsorption, or chronic antibiotic use

Hyperemesis Gravidarum

Vomiting during the first weeks of pregnancy, along with ketosis, and a mild rise in aminotransferase and bilirubin levels Usually resolved by the end of the first trimester

Lysogenic Conversion

When a bacterium acquires a new trait from a virus which has injected its DNA into the bacteria and integrates into the bacterial DNA → this is how E. Coli obtained the Shigalike toxin gene

What is a diagnostic finding on biopsy of advanced cirrhosis?

Wide septa with regenerative hyperplastic nodules

Yellow fever virus Lab findings, Transmission, Disease caused, & Treatment/ Prevention

Yellow Fever: (+) Sense, ssRNA, enveloped virus Transmission: Vector born (Aedes Mosquito) Disease ∙ Toxic Phase: flu-like symptoms, jaundice, bleeding from gums/venipuncture sites which can progress to melena, hematemesis (black vomit) Treatment: Live-attenuated vaccine (should not be given to immunocompromised patients) for prevention, supportive care for treatment

What are the zone of the hepatic lobule?

Zones correspond to the distance from arterial blood supply Zone 1: Least sensitive to hypoxia, Most sensitive to toxins Zone 2: Intermediate zone Zone 3: Most sensitive to hypoxia, Least sensitive to toxins

Indications for GI Endoscopy

• If a change in management is probable based on results of endoscopy • After empirical trial of therapy for suspected benign digestive disorder is unsuccessful • Initial method of evaluation as alternative to radiographic study • When a primary therapeutic procedure is contemplated

What is endoscopic ultrasound (EUS) used for?

• Staging tumors of the GI tract, pancreas, bile ducts, and mediastinum, including lung cancer • Evaluating abnormalities of the GI tract wall or adjacent structures • Tissue sampling of lesions within, or adjacent to, the wall of the GI tract • Evaluation of abnormalities of the pancreas, including masses, pseudocysts, cysts, and chronic pancreatitis.

When is GI Endoscopy not indicated?

• When the results will not contribute to a management choice • For periodic follow-up of healed benign disease unless surveillance of a premalignant condition is warranted

Initial treatment for Acute GI bleeding

∙ 2 Large bore IV's with resuscitation of hemodynamic stability ∙ Blood Transfusions to target hemoglobin >7 g/dL and target INR <2.5 ∙ CBC ∙ Type and Cross ∙ NPO

What bacteria are associated with bloody diarrhea?

∙ Campylobacter ∙ Shigella ∙ EHEC → Also associated with Hemolytic Uremic Syndrome ∙ EIEC ∙ Salmonella

State the risk factors for aspiration pneumonia.

∙ Decayed Teeth (oral bacteria) ∙ Dependence for oral care & eating (dementia) ∙ Number of medications taken ∙ Multiple medical diagnoses ∙ Smoking ∙ Tube feedings: causes reflux that they then aspirate Aspiration pneumonia is caused by oral pathogens

Common sources of lower GI bleeding

∙ Diverticula ∙ Malignancy ∙ Ischemia ∙ Colitis ∙ Hemorrhoids ∙ Angiodysplasia ∙ Unknown

What bacteria are associated with watery diarrhea?

∙ ETEC ∙ Cholera ∙ C. Diff ∙ Cereus ∙ S. Aureus ∙ Giardia ∙ Listeria Monocytogenes ∙ Salmonella

Potential Etiologies of Upper GI Bleeding

∙ Gastric Erosions ∙ Duodenal Ulcers ∙ Gastric Ulcers ∙ Esophageal Varices ∙ Mallory-Weiss Tears ∙ Esophagitis ∙ Rare Causes (Hemobilia, Aorto-enteric Fistula)

What are the normal physiologic changes during pregnancy?

∙ Increase blood volume and cardiac output ∙ Increased ALP 3-4 fold ∙ Increased Clotting factor creating hypercoagulable state ∙ Decreased gallbladder contractility ∙ Decreased Hemoglobin ∙ Decreased Uric acid


Ensembles d'études connexes

Networking - Chapter 3: Network Cabling and Hardware Devices

View Set

Identity Development in Adolescence and Young Adulthood

View Set

Physical Agents Final Exam Review

View Set

Fundamentals of Success Infection Control

View Set

Chapter 17: Freedom's Boundaries, at Home and Abroad Questions

View Set

The Rise of Islam Global History I,Quiz 03-29-22

View Set

Chapter 4 - Life Policy Provisions and Options

View Set